فهرست مطالب

خانواده درمانی کاربردی - سال چهارم شماره 2 (پیاپی 16، تابستان 1402)

نشریه خانواده درمانی کاربردی
سال چهارم شماره 2 (پیاپی 16، تابستان 1402)

  • تاریخ انتشار: 1402/06/05
  • تعداد عناوین: 33
|
  • زهره انصاری، فریبا نادریان، فاطمه گل عابدینی، مریم یاوری کرمانی* صفحات 1-28
    هدف

    هدف از پژوهش حاضر تعیین اثربخشی زوج درمانی هیجان مدار و مقایسه آن با زوج درمانی ساختاری بر عدالت زناشویی و راهبردهای حفظ رابطه در زوجین متعارض بود.

    روش پژوهش:

     پژوهش حاضر نیمه آزمایشی با پیش آزمون و پس آزمون با گروه کنترل و پیگیری دوماهه بود؛ جامعه آماری پژوهش حاضر 205 زوج مراجعه کننده به مراکز مشاوره شهر شیراز در شش ماهه آخر سال 1399 بود که پس از غربالگری با پرسشنامه تعارض زناشویی ثنایی (1387)، 60 زوجی که از نقطه برش 150 نمره بیشتری گرفته بودند به شیوه در دسترس انتخاب و به پرسشنامه های عدالت زناشویی و راهبردهای حفظ رابطه استنفورد، داینتون و هاس (2000) پاسخ دادند و در گروه های آزمایشی زوج درمانی هیجان مدار (10 زوج)، گروه آزمایشی زوج درمانی ساختاری (10 زوج) و گروه گواه (10 زوج) به صورت تصادفی جایگزین شدند. هر دو گروه آزمایش به مدت هشت جلسه 90 دقیقه ای تحت مداخله زوج درمانی هیجان مدار جانسون (2004) و زوج درمانی  ساختاری مینوچین (1974) قرار گرفتند و گروه گواه هیچ مداخله ای را دریافت نکردند. داده ها با استفاده از واریانس مختلط اندازه های مکرر تجزیه وتحلیل شد.

    یافته ها

    نتایج نشان داد زوج درمانی هیجان مدار (97/8=F و 005/0=P) و زوج درمانی ساختاری (04/10=F و 003/0=P)  برافزایش عدالت زناشویی و زوج درمانی هیجان مدار (60/59=F و 001/0=P) و زوج درمانی ساختاری (29/19=F و 001/0=P) بر راهبردهای حفظ رابطه در زوجین متعارض موثر است و این تاثیر در مرحله پیگیری نیز پایدار بود؛ نتایج آزمون تعقیبی نیز نشان داد که تفاوت بین دو گروه زوج درمانی هیجان مدار و زوج درمانی ساختاری در متغیرهای عدالت زناشویی و راهبردهای حفظ رابطه معنادار نیست.

    نتیجه گیری

    یافته ها شواهدی را پیشنهاد می کند که مداخله زوج درمانی هیجان مدار و زوج درمانی ساختاری برای افزایش عدالت زناشویی و راهبردهای حفظ رابطه در همسران دچار تعارض زناشویی روش مناسبی است.

    کلیدواژگان: تعارض، زوج درمانی، هیجان مدار، ساختاری، عدالت زناشویی، راهبردهای حفظ رابطه
  • مهشید نیک نژادی، ایلناز سجادیان*، غلامرضا منشئی صفحات 29-46
    هدف

    هدف این پژوهش تعیین اثربخشی درمان مبتنی برکارآمدی هیجانی بر خودکنترلی و صمیمیت زنان آسیب دیده از ترومای پیمان شکنی همسر بود.

    روش پژوهش: 

    این پژوهش از نوع نیمه آزمایشی با طرح پیش آزمون _ پس آزمون با گروه گواه و پیگیری دو ماهه و جامعه آماری این پژوهش زنان آسیب دیده از پیمان شکنی همسر مراجعه کننده به مراکز مشاوره شهر اصفهان در سال 1399 بود که پس از غربالگری با مقیاس سنجش اختلال پس از آسیب (وددرز و همکاران،1994) 30 نفر به صورت نمونه گیری هدفمند انتخاب و در دو گروه آزمایش و کنترل به صورت تصادفی جایگزین شدند(هرگروه 15 نفر). ابزارهای پژوهش پرسشنامه خودکنترلی (تانجی و همکاران،2004) و صمیمیت (واکر و تامپسون،1983) بود. گروه آزمایش به مدت 8 جلسه 90 دقیقه ای تحت درمان مبتنی برکارآمدی هیجانی مک کی و وست (2016) قرار گرفت، اما گروه گواه تا پایان مرحله پیگیری، هیچ مداخله ای را دریافت نکردند؛ سپس هر دو گروه در سه مرحله پیش آزمون، پس آزمون و پیگیری به وسیله پرسشنامه های خودکنترلی و صمیمیت مورد ارزیابی قرار گرفتند. داده ها با استفاده از تحلیل واریانس اندازه های تکراری تحلیل شد.

    یافته ها

    نتایج حاکی از تاثیر درمان مبتنی برکارآمدی هیجانی بر خودکنترلی (479/22F=، 001/0p<) و صمیمیت (14/57F=، 001/0p<) در مرحله پس آزمون و پایداری این تاثیر در مرحله پیگیری بود.

    نتیجه گیری

    بنابر نتایج این پژوهش برای بهبود خودکنترلی و صمیمیت زنان آسیب دیده از ترومای پیمان شکنی همسر می توان از درمان کارآمدی هیجانی استفاده نمود.

    کلیدواژگان: درمان مبتنی برکارآمدی هیجانی، خودکنترلی، صمیمیت، تروما، پیمان شکنی همسر
  • محمدرضا مهدی قلی، فریده دوکانه ای فرد*، پانته آ جهانگیر صفحات 47-59
    هدف

    زوجین جوان به دلیل نداشتن تجریه و تاب آوری در تعارضات زناشویی، بیشتر در معرض دلزدگی زناشویی و در نتیجه تمایل به طلاق هستند. این پژوهش با هدف تعیین اثربخشی زوج درمانی هیجان مدار بر دلزدگی زناشویی و تاب آوری زوجین جوان انجام شد.

    روش پژوهش:

     از روش تحقیق نیمه آزمایشی با طرح پیش آزمون-پس آزمون با گروه کنترل استفاده شد. از جامعه آماری زوجین جوان مراجعه کننده به مراکز مشاوره خانواده منطقه 1 شهر تهران 30 زوج  (30 مرد و 30 زن) با روش نمونه گیری در دسترس انتخاب و با استفاده از روش تصادفی در دو گروه آزمایش و کنترل جایگزین شدند. ابزارهای پژوهش دلزدگی زناشویی (پاینز، 1996)، تاب آوری (کانر و دیویدسون، 2003) بود. یافته های پژوهش با استفاده از نرم افزار SPSS نسخه 23 و تحلیل واریانس با اندازه گیری مکرر مورد تحلیل قرار گرفت.

    یافته ها

    نتایج نشان داد که زوج درمانی هیجان مدار بر کاهش دلزدگی زناشویی و افزایش تاب آوری زوجین جوان تاثیر معنی دار و پایداری دارد (001/0 p<).

    نتیجه گیری

    بنابراین به منظور بهبود دلزدگی زناشویی و تاب آوری زوجین جوان پیشنهاد می شود از رویکرد زوج درمانی هیجان مدار استفاده شود.

    کلیدواژگان: درمانی هیجان مدار، دلزدگی زناشویی، تاب آوری، زوجین جوان
  • فاطمه ملمع قبادی، الهام صالحی ریزی*، عبدالرضا افزود صفحات 60-78
    هدف

    هدف پژوهش حاضر، بررسی مدل ساختاری تعهد زناشویی بر اساس باورهای غیر منطقی و عدالت زناشویی با میانجی گری تنظیم شناختی هیجان است.

    روش پژوهش: 

    روش پژوهش همبستگی و از نوع مدل سازی معادلات ساختاری بود. جامعه ی آماری در این پژوهش شامل کلیه زوجین مراجعه کننده به مراکز مشاوره تهران در سال 1400 بودند. برای انتخاب نمونه مورد پژوهش از روش نمونه گیری هدفمند استفاده شد، به این صورت که از بین زوجین مراجعه کننده به مراکز مشاوره 300 نفر براساس ملاک های ورود و خروج پژوهش به عنوان گروه نمونه انتخاب شد. ابزار پژوهش شامل پرسشنامه باورهای غیرمنطقی جونز (1968)، پرسشنامه تعهد زناشویی آدامز و جونز (1997)، پرسشنامه تنظیم شناختی هیجان گارنفسکی و کرایج (2006)، مقیاس عدالت زناشویی غفاری و رمضانی (1398) بودند. برای تحلیل داده ها از روش مدل سازی معادلات ساختاری استفاده شد.

    یافته ها

    نتایج آزمون مدل مفهومی نشان داد شاخص های برازش مدل پژوهش در وضعیت مطلوبی قرار دارد. باورهای غیر منطقی بر تعهد زناشویی زوجین اثر غیرمستقیم (به واسطه تنظیم هیجان مثبت و تنظیم هیجان منفی) معنادار دارد (05/0>P). عدالت زناشویی و خودکارآمدی به واسطه ی تنظیم هیجان منفی بر تعهد زناشویی بیماران اثر می گذارند (05/0>P).

    نتیجه گیری

    بر اساس یافته های پژوهش، در مراکز مشاوره، برای افزایش سطح تعهد زناشویی، توجه به پیشایندهای باورهای غیر منطقی و عدالت زناشویی ضروری است.

    کلیدواژگان: تعهد زناشویی، باورهای غیر منطقی، عدالت زناشویی، تنظیم شناختی هیجان
  • زینب کیانوش، بهرام میرزاییان*، حمید نجات صفحات 79-95
    هدف

    هدف پژوهش حاضر مقایسه اثربخشی درمان شناختی- رفتاری و درمان تعامل والد -کودک بر اضطراب و تنهایی کودکان مبتلا به اختلال اضطراب جدایی بود.

    روش پژوهش: 

    پژوهش حاضر از نوع طرح های کاربردی و شبه آزمایشی از نوع پیش آزمون- پس آزمون و پیگیری همراه با گروه گواه بود. جامعه آماری این پژوهش شامل کلیه کودکان (دختر و پسر) مبتلا به اختلال اضطراب جدایی شهر ساری در شش ماه نخست سال 1400 بود. نمونه این پژوهش شامل 45 نفر از مراجعه کننده به مرکز مشاوره بامداد و مرکز مشاوره باور که با استفاده از روش نمونه گیری در دسترس انتخاب شدند و به طور تصادفی در دو گروه آزمایشی درمان شناختی- رفتاری و درمان تعامل والد -کودک و گروه گواه در لیست انتظار جای داده شدند. داده ها با استفاده از پرسشنامه، مقیاس اضطراب کودکان اسپنس (1997) و پرسشنامه تنهایی آشر و همکاران (1984) به دست آمد. داده ها با استفاده از روش تحلیل واریانس با اندازه گیری مکرر و نرم افزارSPSS-26 به اجرا درآمد.

    یافته ها

    نتایج نشان داد که بین دو گروه درمان شناختی-رفتاری و درمان تعامل والد -کودک بر اضطراب و تنهایی در کودکان مبتلا به اختلال اضطراب جدایی تفاوت معناداری وجود دارد و درمان درمان شناختی- رفتاری سبب ایجاد تغییرات بیشتر در اضطراب در کودکان مبتلا به اختلال اضطراب جدایی شد و این درمان قویتر از درمان درمان تعامل والد-کودک در این گروه از بیمارن است. در درمان تعامل والد -کودک سبب ایجاد تغییرات بیشتر در تنهایی در کودکان مبتلا به اختلال اضطراب جدایی شد و این درمان قوی تر از درمان شناختی- رفتاری در این گروه از کودکان مبتلا به اختلال اضطراب جدایی بود (001/0>P).

    نتیجه گیری

    میتوان نتیجه گرفت که درمان شناختی-رفتاری بر اضطراب در کودکان مبتلا به اختلال اضطراب جدایی و درمان تعامل والد -کودک می تواند به عنوان روش مداخله ای موثری برای بهبود تنهایی کودکان مبتلا به اختلال اضطراب جدایی باشد.

    کلیدواژگان: درمان شناختی- رفتاری، درمان تعامل والد -کودک، اضطراب، تنهایی، اختلال اضطراب جدایی
  • اعظم سرآبادانی، رمضان حسن زاده*، حسینعلی قنادزادگان صفحات 96-114
    هدف

    پژوهش حاضر با هدف اثربخشی درمان مبتنی بر پذیرش و تعهد بر تحمل پریشانی و تنظیم شناختی هیجان زنان دارای اختلال اضطراب فراگیر می باشد.

    روش پژوهش: 

    از نوع نیمه آزمایشی با طرح پیش آزمون- پس آزمون با گروه گواه (یک گروه آزمایشی و یک گروه گواه) و پیگیری سه ماهه بود. جامعه آماری زنان دارای اختلال اضطراب فراگیر مراجعه کننده به مراکز مشاوره و خانه های سلامت مناطق 1، 2، 3 و 5 شهر تهران در نیمه اول سال 1400 بود. نمونه شامل 30 زن دارای اختلال اضطراب فراگیر بودند که با آزمون اضطراب فراگیر و دارا بودن ملاک های ورودی به صورت هدفمند و در دسترس انتخاب شدند. برای جمع آوری داده ها از ابزارهای؛ ""مقیاس تحمل پریشانی"، "پرسشنامه تنظیم شناختی هیجان" و "مقیاس اضطراب فراگیر"، استفاده شد. گروه آزمایش مداخله درمان مبتنی بر پذیرش و تعهد را طی دو ماه در 8 جلسه 90 دقیقه ای دریافت نمودند. این در حالی بود که گروه گواه این مداخله را در طول فرایند انجام پژوهش دریافت نکرد و یک دوره پیگیری بعد از سه ماه انجام گرفت. داده های جمع آوری شده در نرم افزار اس پی اس اس نسخه 27 تحلیل شد.

    یافته ها

    نتایج نشان داد درمان پذیرش و تعهد به صورت معنادار تحمل پریشانی را بالا برده، راهبردهای انطباقی تنطیم شناختی هیجان را به صورت معنادار افزایش و راهبردهای غیر انطباقی تنظیم شناختی را به صورت معنادارکاهش داده است و اختلاف معنا داری در هر دو متغیر بین پیش آزمون و پس آزمون و همچنین بین پیش آزمون و دوره پیگیری وجود داشت.

    نتیجه گیری

    درمان مبتنی بر پذیرش و تعهد بر زنان دارای اختلال اضطراب فراگیر تاثیر می گذارد و تحمل پریشانی و تنظیم شناختی هیجان را در آنها بهبود می بخشد. توصیه می شود درمان مبتنی بر پذیرش و تعهد برای زنان دارای اختلال اضطراب فراگیر به کار گرفته شود.

    کلیدواژگان: پذیرش و تعهد، تحمل پریشانی، تنظیم شناختی هیجان، اختلال اضطراب فراگیر
  • شیوا وطن پناه، جواد خلعتبری*، افشین طیبی، مهرداد ثابت صفحات 115-128
    هدف

    پژوهش حاضر، با هدف تعیین اثربخشی درمان متمرکز بر شفقت بر رفتار خوردن هیجانی، بدتنظیمی هیجانی، استرس ادراک شده و نشخوار فکری در زنان مبتلا به بیماری چاقی مزمن انجام شد.

    روش پژوهش:

     پژوهش حاضر جزء طرح های نیمه آزمایشی با پیش آزمون -پس آزمون-پیگیری با گروه گواه بود. جامعه آماری پژوهش حاضر، شامل کلیه زنان مبتلا به بیماری چاقی مزمن و شاخص توده بدنی مراجعه کننده به کلینیک مشاوره تغذیه ای کیان شهر تهران بین ماه های اردیبهشت تا تیرماه 1401 بودند. به منظور تشکیل دو گروه با استفاده از روش نمونه گیری هدفمند (با در نظر گرفتن ملاک های ورود و خروج)، ابتدا 30 زنان مبتلا به چاقی مزمن و شاخص توده بدنی به شیوه غربالگری انتخاب و سپس به صورت تصادفی 15 بیمار در گروه آزمایش و 15 بیمار در گروه گواه گروه بندی شدند. برنامه جلسات درمان متمرکز بر شفقت در طی 8 جلسه 90 دقیقه ای براساس پروتکل نف و گرمر (2013) برای گروه آزمایش تشکیل شد. همچنین مرحله پیگیری سه ماه بعد از پس آزمون روی هر سه گروه اجرا شد. ابزار پژوهش شامل پرسشنامه رفتار خوردن هیجانی ون استرین و همکاران (2009)، پرسشنامه بدتنظیمی هیجانی گراتز و رومر (2004)، پرسشنامه استرس ادراک شده کوهن و همکاران (1983) و پرسشنامه نشخوار فکری نالن هوکسما و مارو (1991) بودند. داده های پژوهش با استفاده از آزمون تحلیل واریانس اندازه گیری مکرر تحلیل شدند.

    یافته ها

    نتایج تحلیل داده ها نشان داد درمان متمرکز بر شفقت در شرکت کنندگان گروه های آزمایشی به نسبت گروه گواه تاثیر معناداری بر بهبود رفتار خوردن هیجانی (83/115=F، 001/0>P)، بدتنظیمی هیجانی(10/98=F، 001/0>P)، استرس ادراک شده (53/101=F، 001/0>P) و نشخوار فکری (52/74=F، 001/0>P) در زنان مبتلا به بیماری چاقی مزمن و شاخص توده بدنی داشته است (05/0>P).

    نتیجه گیری

    بنابراین می توان نتیجه گرفت درمان متمرکز بر شفقت باعث بهبود رفتار خوردن هیجانی، بدتنظیمی هیجانی، استرس ادراک شده و نشخوار فکری در زنان مبتلا به چاقی مزمن می گردد.

    کلیدواژگان: شفقت، رفتارخوردن هیجانی، بدتنظیمی هیجانی، استرس ادراک شده، نشخوار فکری
  • آمنه حسن پور، ابراهیم علیزاده موسوی*، محمد محمدی پور صفحات 129-142
    هدف

    هدف پژوهش حاضر تعیین نقش میانجی نشخوار فکری بین کنترل والدینی با کمرویی دختران نوجوان بود.

    روش پژوهش: 

    این پژوهش توصیفی- پیمایشی از نوع همبستگی است. هدف تحقیق کاربردی و از نظر زمان مقطعی می باشد. جامعه آماری، شامل کلیه ی دانش آموزان دختر متوسطه دوم شهر بجنورد است که در سال تحصیلی 1399-1400 مشغول به تحلیل بوده اند. جهت برآورد حجم نمونه کافی از روش تعیین حجم نمونه کلاین (2005) استفاده شد که تعداد 567 نفر به عنوان حجم نمونه انتخاب شد. روش نمونه گیری خوشه ای تک مرحله ای می باشد. به منظور جمع آوری اطلاعات موردنظر و سنجش متغیرهای تحقیق، از پرسشنامه نشخوار فکری توسط هوکسما و مارو در سال (1991) کنترل والدینی (PLOC) توسط لسلی ک. کمپیز، رابرت دی. لایمن و استیون پرنتیس دان (1986) و پرسشنامه کمرویی استنفورد استفاده شد و جهت تجزیه و تحلیل اطلاعات از نرم افزار spss و Amose استفاده شد.

    یافته ها

    نتایج، مدل علی کمرویی دختران نوجوان بر اساس کنترل والدین با نقش میانجی نشخوار فکری دارای برازش مناسب است. کنترل والدین بر خود انتقادی تاثیر مستقیم دارد؛ همچنین کنترل والدین بر کمرویی و برن شخوار فکری تاثی رمستقیم دارد. همچنین نشخوار فکری بر کمرویی تاثیرمستقیم دارد.

    نتیجه گیری

    با توجه به نتایج پژوهش می توان گفت که نشخوار فکری نقش میانجی بین کنترل والدین و کمرویی ایفا می کند.

    کلیدواژگان: نشخوار فکری، کنترل والدینی، کمرویی، دختران نوجوان
  • الکسیس گریگوراش، امان تادروس *، انها ژورمالانی صفحات 143-155
  • میترا محمدیان خوانساری، علی اصغر حسین زاده* صفحات 156-172
    هدف
    پژوهش حاضر با هدف پیش بینی نگرش جوانان مجرد به ازدواج بر اساس صفات شخصیتی و کمال گرایی انجام شد.
    روش
    پژوهش حاضر از نظر هدف کاربردی و طرح پژوهش توصیفی از نوع همبستگی بود، جامعه آماری شامل کلیه دانشجویان مجرد دانشگاه های شهر اصفهان در نیمسال دوم سال تحصیلی 1399-1400 بود ند که از جامعه مزبور تعداد 170 نفر به روش در دسترس انتخاب شدند و به پرسشنامه های نگرش نسبت به ازدواج براتن و روزن (1998)، پرسشنامه 5 عامل بزرگ شخصیت فرم کوتاه (NEO-FFI)، پرسشنامه استاندارد کمال گرایی تری - شورت و همکاران (1995) پاسخ دادند. برای تجزیه و تحلیل داده ها از روش همبستگی پیرسون و تحلیل رگرسیون سلسه مراتبی با استفاده از نرم افزار SPSS استفاده شد.
    یافته ها
    نتایج نشان داد از بین صفات شخصیتی، نوروزگرایی به صورت منفی و برونگرایی به صورت مثبت و معنادار نگرش جوانان مجرد به ازدواج را پیش بینی می کند. همچنین، کمال گرایی منفی بطور منفی و کمال گرایی مثبت به صورت مثبت و معنی دار قادر به پیش بینی نگرش جوانان مجرد به ازدواج بود.
    نتیجه گیری
    بر اساس یافته های پژوهش می توان چنین نتیجه گرفت که صفات شخصیتی و کمال گرایی از عواملی هستند که در نگرش جوانان به ازدواج نقش دارند.
    کلیدواژگان: نگرش به ازدواج، صفات شخصیتی، کمال گرایی
  • مجتبی رئیسی سرتشنیزی*، آمنه رئیسی سرتشنیزی صفحات 173-185
    هدف
    هدف این پژوهش بررسی تاثیر مشاوره زوجی هیجان مدار بر تصویر خود تهدید شده بود.
    روش
    روش این تحقیق نیمه تجربی و از نوع پیش آزمون-پس آزمون با گروه کنترل بود. جامعه آماری این پژوهش شامل زوجین مراجعه کننده به فرهنگسرای خانواده و مراکز مشاوره شهر اصفهان بود. برای انتخاب نمونه موردنظر 40 زوج (80 نفر) به صورت داوطلب انتخاب و به صورت تصادفی در دو گروه آزمایش و گواه (20 زوج در گروه آزمایش و 20 زوج در گروه کنترل) قرار گرفتند. ابزار پژوهش تصویر خود تهدید شده از پرسشنامه NI-90 بود. متغیر مستقل مشاوره زوجی هیجان مدار بود که در 10 جلسه روی گروه آزمایش اعمال شد.
    یافته ها
    نتایج حاصل از تحلیل کوواریانس نشان داد که مشاوره زوجی هیجان مدار بر تصویر خود تهدید شده اثر داشت ولی بر ابعاد امید، مسخ واقعیت، خروج به احساس هماهنگی، متغیر خود تهدید شده اثر نداشت.
    نتیجه گیری
    بنابراین می توان نتیجه گرفت مشاوره زوجی هیجان مدار موجب کاهش تصویر خودتهدیدشده زوجین می شود.
    کلیدواژگان: مشاوره زوجی هیجان مدار، هیجانات منفی، تصویر خود تهدیدشده
  • الهام خواجه نظامی، ویدا سادات رضوی*، ویدا اندیشمند، امان الله سلطانی صفحات 186-204
    هدف

    هدف پژوهش حاضر، طراحی و اعتبارسنجی بسته آموزشی مدیریت استرس و خشم بر مبنای روش کتاب درمانی و بازی درمانی در کودکان بود.

    روش پژوهش: 

    این پژوهش چند وجهی است، کندو کاو در این پدیده چند وجهی محقق را به انتخاب روش تحقیق آمیخته کیفی-کمی) سوق داد. از میان روش های تحقیق آمیخته از طرح تحقیق آمیخته اکتشافی متوالی استفاده خواهد شد. به منظور جمع آوری داده ها، علاوه بر استفاده از مطالعات کتابخانه ای و مبانی نظری بر گرفته از تحلیل محتوا، از مصاحبه عمیق نیمه ساختار یافته برای درک بیشتر استفاده خواهد شد.

    یافته ها

    نتایج این بررسی نشان می دهد که پکیج های بازی درمانی، قصه درمانی می تواند مورد استفاده روان شناسان در درمان کردن استرس و خشم در کودکان قرار گیرد. نتایج حاصل همچنین نشان داد میانگین پرخاشگری و استرس پس از استفاده از بازی درمانی و کتاب درمانی، به طور معناداری کاهش داشته است.

    نتیجه گیری

    یافته های به دست آمده از این پژوهش اشاره به این موضوع دارد که استفاده از این روش ها، منجر به کاهش استرس کودکان و افزایش پذیرش آنها در جامعه می شود.

    کلیدواژگان: بسته آموزشی، مدیریت استرس و خشم، کتاب درمانی، بازی درمانی، کودکان
  • پریسا سبزواری، غلامرضا خویی نژاد*، محمدرضا صفاریان طوسی صفحات 205-226
    هدف

    پژوهش حاضر با هدف مقایسه اثربخشی رویکرد خانواده محور بوین و خانواده درمانی ساختاری مینوچین بر بهبود الگوهای ارتباطی و کاهش تعارضات زناشویی زنان دچار طلاق عاطفی صورت پذیرفت.

    روش پژوهش: 

    روش پژوهش تجربی از نوع پیش آزمون-پس آزمون همراه با گروه کنترل بود. جامعه آماری کلیه زنان دچار طلاق عاطفی مراجعه کننده به سازمان مرکزی بهزیستی شهر اصفهان در بازه زمانی سه ماهه اول سال 1399 بودند که پس از غربال گری، 45 آزمودنی دچار طلاق عاطفی، به روش تصادفی ساده انتخاب و در دو گروه آزمایش و کنترل نامعادل گمارده شدند. ابزار پژوهش مقیاس الگوهای ارتباطی و تعارضات زناشویی بود. آزمودنی های گروه های آزمایش در برنامه آموزشی خانواده درمانی بوین و مینوچین شرکت کردند، اما گروه کنترل هیچ آموزشی دریافت نکردند. درنهایت داده ها با استفاده از نرم افزار SPSS و در دو سطح آمار توصیفی و آمار استنباطی مورد تجزیه وتحلیل قرار گرفت.

    یافته ها

    بین اثربخشی دو گروه آزمایش (بوین و مینوچین) بر الگوهای ارتباطی زنان دچار طلاق عاطفی، تفاوت معنی داری وجود دارد و مقایسه میانگین ها حاکی از عملکرد بهتر خانواده درمانی ساختاری مینوچین بود؛ و بین اثربخشی دو گروه آزمایش (بوین و مینوچین) بر تعارضات زناشویی زنان دچار طلاق عاطفی، تفاوت معنی داری وجود ندارد و تنها در بعد کاهش ارتباط با خویشاوندان حاکی از اثربخشی بهتر رویکرد ساختاری مینوچین بود.

    نتیجه گیری

    رویکرد درمانی بوین و مینوچین می توانند در کاهش میزان تعارضات زناشویی و بهبود الگوی ارتباطی زنان دچار طلاق عاطفی سودمند باشد و متعاقبا، طلاق عاطفی را کاهش و سازگاری زناشویی را افزایش دهد.

    کلیدواژگان: خانواده درمانی، الگوهای ارتباطی، تعارضات زناشویی، طلاق عاطفی
  • فاطمه حیدری، حسن احدی*، بیوک تاجری صفحات 227-240
    هدف

    زنان نابارور در زندگی با اضطراب زیادی مواجه می شوند و به نظر می رسد که درمان های برگرفته از موج سوم روان درمانی بتواند در کاهش اضطراب آنها موثر واقع شود. در نتیجه، هدف این مطالعه مقایسه اثربخشی درمان مبتنی بر شفقت و درمان شناختی مبتنی بر ذهن آگاهی بر اضطراب زنان نابارور بود.

    روش پژوهش: 

    پژوهش حاضر از نظر هدف کاربردی و از نظر شیوه اجرا نیمه تجربی با طرح پیش آزمون، پس آزمون و پیگیری دو ماهه با گروه گواه بود. جامعه پژوهش زنان نابارور مراجعه کننده به کلینیک های دارای پزشک متخصص باروری شهر تهران در سال 1401 بودند. نمونه پژوهش 45 نفر بودند که پس از بررسی معیارهای ورود به پژوهش با روش نمونه گیری هدفمند انتخاب و به صورت تصادفی در سه گروه مساوی شامل دو گروه آزمایش و یک گروه گواه جایگزین شدند. هر یک از گروه های آزمایش به تفکیک 8 جلسه 90 دقیقه ای تحت درمان های مبتنی بر شفقت و شناختی مبتنی بر ذهن آگاهی قرار گرفتند و در این مدت گروه گواه درمانی دریافت نکرد. داده ها با سیاهه اضطراب بک و همکاران (1988) گردآوری و با روش های تحلیل واریانس آمیخته و آزمون تعقیبی بونفرونی در نرم افزار SPSS تحلیل شدند.

    یافته ها

    تحلیل ها نشان داد که هر دو روش درمان مبتنی بر شفقت و درمان شناختی مبتنی بر ذهن آگاهی در مقایسه با گروه گواه باعث کاهش اضطراب زنان نابارور شدند و اثربخشی درمان مبتنی بر شفقت در مقایسه با درمان شناختی مبتنی بر ذهن آگاهی در کاهش اضطراب آنان بیشتر بود (001/0>P).

    نتیجه گیری

    طبق نتایج پژوهش حاضر و با توجه به اثربخشی بیشتر درمان مبتنی بر شفقت در مقایسه با درمان شناختی مبتنی بر ذهن آگاهی در کاهش اضطراب زنان نابارور، پیشنهاد می شود که درمانگران در مداخله های خود از روش درمان مبتنی بر شفقت استفاده نمایند.

    کلیدواژگان: درمان مبتنی بر شفقت، درمان شناختی مبتنی بر ذهن آگاهی، اضطراب، زنان نابارور
  • صدیقه افشاری، سید عبدالمجید بحرینیان*، فاطمه شهابی زاده صفحات 241-257
    هدف

    هدف پژوهش حاضر، تعیین اثربخشی آموزش مهارت های اجتماعی هیجانی بر باورهای انگیزشی و راهبردهای یادگیری خودتنظیمی دختران مبتلا به اختلال یادگیری بوده است.

    روش پژوهش: 

    روش تحقیق حاضر از لحاظ هدف، کاربردی و از نظر موقعیت میدانی و از نظر روش نیمه آزمایشی با طرح پیش‎آزمون، پس‎آزمون و پیگیری با گروه گواه و انتصاب تصادفی بوده است. جامعه آماری این پژوهش، شامل تمام دختران 10 تا 13 سال دارای اختلال یادگیری خواندن، نوشتن و ریاضی شهرستان جویبار در سال تحصیلی 1400-1399 بوده است. نمونه گیری به صورت هدفمند و 30 نفر از دختران به عنوان نمونه انتخاب شدند و سپس با گمارش تصادفی، 15 نفر به گروه آموزش مهارت های اجتماعی هیجانی و 15 نفر به گروه گواه تخصیص داده شدند. گروه آزمایش به مدت 10 جلسه تحت آموزش مهارت های اجتماعی هیجانی براساس مدل اسکویرز (2003) قرار گرفتند؛ در حالی که گروه گواه هیچ مداخله ای را دریافت نکرد. همچنین پرسشنامه باورهای انگیزشی هارتر (1981)، و پرسشنامه یادگیری خودتنظیمی ریان و کانل (1989) در مرحله پیش آزمون، پس آزمون و پیگیری مورد استفاده قرار گرفت. در نهایت، داده ها با استفاده از آزمون تحلیل واریانس با اندازه گیری مکرر و آزمون تعقیبی بونفرونی مورد تجزیه و تحلیل قرار گرفت.

    یافته ها

    نتایج نشان داد که آموزش مهارت های اجتماعی هیجانی بر باورهای انگیزشی (95/16=F؛ 001/0=P) و راهبردهای یادگیری خودتنظیمی (57/88=F؛ 001/0=P) دختران تاثیر دارد.

    نتیجه گیری

    می توان نتیجه گرفت آموزش مهارت های اجتماعی هیجانی بر باورهای انگیزشی و راهبردهای یادگیری خودتنظیمی دختران موثر است.

    کلیدواژگان: مهارت های اجتماعی هیجانی، باورهای انگیزشی، یادگیری خودتنظیمی، اختلال یادگیری
  • سعیده فرج نیا، محمدرضا بلیاد*، سیمین حسینیان، مرجان حسین زاده تقوایی صفحات 258-278
    هدف

    یکی از مهم ترین رابطه ها بین یک زن و یک مرد، ازدواج است که شامل هم تعهد عاطفی و هم قانونی می شود و از اهمیت ویژه ای در زندگی هر فرد بزرگ سال برخوردار است. بنابراین هدف این پژوهش مدل یابی سازگاری زناشویی بر اساس الگوهای ارتباطی و عملکرد رابطه زناشویی با میانجی گری نگرش معنوی بود.

    روش پژوهش: 

    پژوهش حاضر توصیفی- همبستگی از نوع مدل یابی معادلات ساختاری بود. جامعه آماری پژوهش شامل افراد متاهل ساکن شهر تهران بودند که با توجه به تعداد تغییرهای مشاهده پذیر، به روش در دسترس 350 نفر نمونه به پرسشنامه های سازگاری زناشویی اسپانیر(1976)، نگرش معنوی: شهیدی و فرج نیا (1391)، الگوهای ارتباطی کریستنسن و سالاوی (1984) و عملکرد جنسی: فرج نیا و همکاران(1393) پاسخ دادند. داده های پژوهش نیز از طریق نرم افزار SPSS و AMOS مورد تجزیه تحلیل قرار گرفت.

    یافته ها

    یافته ها نشان داد که الگوی ارتباطی سازنده به طور مستقیم به میزان 334/0 = β از واریانس سازگاری زناشویی را تبیین می کند و الگوی ارتباطی کناره گیری نیز به طور مستقیم به میزان 159/0 - = β از واریانس سازگاری زناشویی را تبیین می کند. همچنین الگوی ارتباطی سازنده به طور مستقیم به میزان 21/0= β از واریانس نگرش معنوی را تبیین می کند و نگرش معنوی  به طور مستقیم به میزان 147/0= β از واریانس سازگاری زناشویی را تبیین می کند.

    نتیجه گیری

    می توان نتیجه گرفت که مدل یابی سازگاری زناشویی بر اساس الگوهای ارتباطی و عملکرد رابطه زناشویی با میانجی گری نگرش معنوی دارای اعتبار است.

    کلیدواژگان: سازگاری زناشویی، الگوهای ارتباطی، عملکرد جنسی، نگرش معنوی
  • مجید محمودی مظفر* صفحات 279-290
    هدف

    ازدواج باتوجه به عوامل مختلف ازجمله مشکلات بین فردی زوجین می تواند پایدار یا ناپایدار باشد. هدف پژوهش، تعیین اثربخشی آموزش بسته مشاوره پیش از ازدواج مبتنی بر تلفیق رویکردهای طرحواره درمانی و ایماگوتراپی بر درمان مشکلات بین فردی زوجین در آستانه ازدواج بود.

    روش پژوهش: 

    پژوهش به روش نیمه آزمایشی با طرح پیش آزمون،پس آزمون و پیگیری دوماهه با گروه گواه بود. جامعه آماری را دختر و پسرهای در آستا نه ازدواج مراجعه کننده به مرکز مشاوره راه فردا در شهر تهران در سال 1400 تشکیل دادند که بیست زوج به روش نمونه گیری تصادفی انتخاب شدند. سپس به طور تصادفی در دو گروه متشکل از 10 زوج آزمایش و 10 زوج گواه قرار گرفتند. برای هر دو گروه، پیش آزمون، پس آزمون و پیگیری اجرا شد.پرسشنامه مشکلات بین فردی(هرویتز و همکاران، 1989) به کار رفت. بسته مشاوره در هشت جلسه نود دقیقه ای دو بار در هفته صرفا برای گروه آزمایش اجراشد. تحلیل داده ها با استفاده از روش تحلیل واریانس با اندازه گیری مکرر و آزمون تعقیبی LSD در نرم افزار SPSS نسخه 22 صورت گرفت. سطح معناداری آزمون های آماری 05/0 در نظر گرفته شد.

    یافته ها

    نتایج نشان داد، اثر اصلی زمان (006/0= p) اثر اصلی گروه (001/0 = P) و اثر متقابل زمان و گروه (001/0 = P) بر متغیر مشکلات بین فردی معنادار بود. در گروه آزمایش، میانگین نمرات پیش آزمون متغیر مشکلات بین فردی زوجین به طور معناداری بیشتر از میانگین نمرات پس آزمون (015/0 = P) و پیگیری (024/0 = P) بود. به علاوه، تفاوت معناداری بین میانگین نمرات متغیر مذکور بین مراحل پس آزمون و پیگیری مشاهده نشد  (228/0 = p)که به معنای ماندگاری اثر مداخله آموزش ها، در طول زمان در گروه آزمایش است.

    نتیجه گیری

    براساس نتایج، آموزش بسته تدوین شده مشاوره پیش از ازدواج برمبنای تلفیق رویکردها و مفاهیم طرحواره درمانی و ایماگوتراپی، بر کاهش مشکلات بین فردی زوجین در آستانه ازدواج اثربخشی دارد.

    کلیدواژگان: بسته مشاوره پیش از ازدواج، مشکلات بین فردی، زوجین در آستانه ازدواج
  • مرتضی حیدری*، زینب سادات میرشعبانی، عاطفه صادق مسجدی، حانیه مرتضایی صفحات 291-303
    هدف

    پژوهش حاضر جهت بررسی اثربخشی گروه درمانی ذهن اگاهی مبتنی بر شناخت بر کاهش اضطراب و افزایش کیفیت زندگی زوجین دارای شوهربزهکار انجام گرفت.

    روش پژوهش: 

    طرح پژوهش حاضر شبه آزمایشی از نوع پیش آزمون- پس آزمون و پیگیری با گروه گواه بود. جامعه اماری شامل کلیه بزهکاران و همسرانشان بود که در طول سال 1400 به کلینیک خدمات روانشناسی و مشاوره وصال مراجعه می نمایند. برای نمونه گیری، از روش نمونه گیری در دسترس استفاده می شود که 20 نفر از زوجین، از مراجعان کلینیک خدمات روانشناسی و مشاوره وصال انتخاب و به صورت تصادفی در گروه ازمایش و گروه گواه جایگزین می شوند. اعضای گروه آزمایش 8 جلسه گروه درمانی ذهن اگاهی مبتنی بر شناخت را دریافت کردند و به اعضای گروه گواه هیچ مداخله ای ارایه نشد. قبل و بعد از اجرای مداخله پرسشنامه اضطراب بک (بک و همکاران، 1990) و کیفیت زندگی (گروه سازمان جهانی بهداشت، 1998) توسط گروه نمونه تکمیل شدند.

    یافته ها

    نتایج تحلیل واریانس اندازه گیری مکرر نشان داد که در مراحل پس آزمون و پیگیری اضطراب زنان دارای شوهربزهکار گروه آزمایش نسبت به گروه گواه کاهش معناداری (74/89 =F، 001/0 =P) و کیفیت زندگی گروه ازمایش نسبت به گروه گواه افزایش (31/110 =F، 001/0 =P) معنادار یافته است.

    نتیجه گیری

    یافته های پژوهش موید این مهم است که گروه درمانی ذهن آگاهی مبتنی بر کاهش اضطراب باعث کاهش تعارض های زناشویی زوجین دارای همسر بزهکار می گردد.

    کلیدواژگان: ذهن اگاهی، اضطراب، کیفیت زندگی، زنان دارای شوهربزهکار
  • ریحانه فخرزارع، مجتبی انصاری شهیدی*، ندا ادیبی صفحات 304-314
    هدف

    آکنه به عنوان یکی از عوامل اصلی ناتوانی در جهان، اثرات منفی روانی و اجتماعی عمیقی را ایجاد می کند. هدف این مطالعه، بررسی رابطه کیفیت زندگی و بیماری آکنه است.

    روش پژوهش:

     این مطالعه از روش توصیفی از نوع پیش بینی استفاده کرده است. 93 نفر در این آزمون شرکت کردند که در بازه سنی 15 تا 50 سال بودند و با روش نمونه‎گیری در دسترس وارد پژوهش شدند. برای شدت ابتلا از مقیاس درجه بندی GAGS استفاده شد. برای تعیین میزان کیفیت زندگی افراد مبتلا به آکنه از پرسش نامه کیفیت زندگی پوستی (DLQI) استفاده گردید.

    یافته ‎ها:

     کیفیت زندگی در میان گروه های شدت آکنه نرمال نبود، بنابراین برای محاسبه همبستگی از آزمون کروسکال- والیس استفاده گردید که بیان کرد رابطه ای معنی دار میان این دو متغیر برقرار است (001/0 < P).

    نتیجه‎ گیری:

     کیفیت زندگی و آکنه ولگاریس همبستگی بسیار زیادی دارند. باتوجه به فراوانی این اختلال و تاثیر آن بر عملکرد اجتماعی و عاطفی افراد، لزوم بررسی بیشتر جهت پیشگیری و درمان این اختلال نیز امری ضروری است.

    کلیدواژگان: آکنه، کیفیت زندگی، اختلالات روان شناختی
  • مینا دلداده مهربان، رمضان حسن زاده*، بهرام میرزاییان صفحات 315-323
    هدف

    پژوهش حاضر با هدف بررسی مقایسه اثربخشی رویکرد زوج درمانی مبتنی بر طرحواره و اکت بر دلزدگی زناشویی، تنظیم هیجان در پرستاران دارای تعارضات زناشویی انجام شد.

    روش پژوهش: 

    روش پژوهش حاضر، آزمایشی است. طرح پژوهش نیمه آزمایشی با طرح پیش آزمون - پس آزمون و پیگیری یک ماهه با گروه گواه است. جامعه آماری شامل تمامی پرستاران مشغول به خدمت در بیمارستان دولتی امام سجاد شهرستان رامسر در سال 1399 بود. از این بین تعداد 30 زوج (90 نفر) به روش نمونه گیری غیرتصادفی در دسترس انتخاب و در گروه آزمایشی (زوج درمانی مبتنی بر طرحواره و اکت) و یک گروه گواه به صورت تصادفی ساده جایگزین شدند (15 زوج در هر گروه). سپس 10 جلسه 90 دقیقه ای (هفته ای 2 جلسه) زوج درمانی مبتنی بر طرحواره و اکت (لیو و مک کی، 2017) در گروه های آزمایشی انجام شد. داده ها با استفاده از پرسشنامه تعارضات زناشویی ثنایی و براتی (1379)، پرسشنامه های دلزدگی زناشویی پاینز (1996) و تنظیم هیجان گراس و جان (2003) جمع آوری شد. داده ها با استفاده از نرم افزار SPSS-24 و از طریق آزمون تحلیل واریانس با اندازه گیری مکرر تحلیل شد.

    یافته ها

    نتایج پژوهش نشان داد که زوج درمانی مبتنی بر طرحواره و اکت بر دلزدگی زناشویی (001/0>P، 53/13=F) و تنظیم هیجان (001/0>P، 86/49=F) در پرستاران دارای تعارضات زناشویی اثربخش است.

    نتیجه گیری

    باتوجه به اثربخشی زوج درمانی مبتنی بر طرحواره و اکت در پرستاران دارای تعارضات زناشویی، پیشنهاد می شود درمانگران از این رویکرد درمانی و آموزشی استفاده نمایند.

    کلیدواژگان: طرحواره درمانی، درمان پذیرش و تعهد، دلزدگی زناشویی، تنظیم هیجان
  • سیمین سلیمانی، لیدا لیل آبادی*، مرجان جعفری روشن صفحات 324-339
    هدف

    پژوهش حاضر با هدف مقایسه اثر بخشی درمان امید محور و درمان مبتنی بر پذیرش و تعهد بر افسردگی و اضطراب مرگ بیماران مبتلا به دیابت نوع دو بود.

    روش پژوهش: 

    پژوهش حاضر یک طرح نیمه آزمایشی پیش آزمون و پس آزمون با گروه گواه و پیگیری بود که تعداد 45 نفر از بیماران مبتلا به دیابت نوع دو مراجعه کننده به انجمن دیابت ایران که واجد ملاک های ورود به پژوهش بودند با روش نمونه گیری در دسترس انتخاب و به صورت تصادفی در دو گروه کنترل و آزمایش قرار گرفتند، برای گروه آزمایش، درمان امید محور و درمان مبتنی بر پذیرش و تعهد اجرا شد اما گروه کنترل هیچ نوع درمانی دریافت نکرد. هر دو گروه پیش از انجام مداخله و پس از آن پرسش نامه افسردگی (بک و همکاران، 1996) و اضطراب مرگ (تمپلر، 1970) را تکمیل کردند. داده ها با استفاده از آزمون تحلیل واریانس آمیخته مورد تجزیه و تحلیل قرار گرفت.

    یافته ها

    نتایج نشان داد اثربخشی درمان امید محور و درمان مبتنی بر پذیرش و تعهد باعث کاهش افسردگی و اضطراب مرگ بیماران مبتلا به دیابت نوع دو شده است و در تفاوت اثر بخشی بین دو درمان، درمان مبتنی بر پذیرش و تعهد نسبت به درمان امید محور موجب کاهش افسردگی بهتر با اثربخشی بالاتر نسبت به درمان امید محور در بیماران مبتلا به دیابت نوع دو شد (05/0=p).

    نتیجه گیری

    نتایج این پژوهش پیشنهاد می کند در کنار پیروی از درمان های دارویی از درمان مبتنی بر پذیرش و تعهد و درمان امید محور جهت بهبود افسردگی و اضطراب مرگ در بیماران مبتلا به دیابت نوع دو می توان استفاده کرد تا به این وسیله از عوارض بیماری، پیشگیری به عمل آید.

    کلیدواژگان: افسردگی، اضطراب مرگ، درمان امید محور، درمان پذیرش و تعهد، دیابت
  • ساره خدایی، زهرا یوسفی*، فلور خیاطان صفحات 340-355
    هدف

    امروزه توجه طلبی به موضوعی نگران کننده در بافت خانواده تبدیل شده است و لزوم پیشگیری و تعدیل آن به عنوان یک ضرورت درمانی احساس می شود. از این رو پژوهش حاضر با هدف واکاوی مولفه های مدیریت موثر توجه طلبی در بین زنان متاهل از منظر مشاوران خانواده و ارایه ی یک مدل درمانی انجام شد.

    روش پژوهش: 

    محیط پژوهش کلیه ی مشاوران خانواده در سال 1400-1401 بود. از بین مشاوران با تجربه که در زمینه خانواده درمانی تجربه لازم را داشتند با استفاده از روش نمونه گیری هدف مند 9 نفر برای مشارکت در پژوهش انتخاب شدند و مورد مصاحبه قرار گرفتند. روش مطالعه تحلیل مضمون تعمقی بود. ابزار پژوهش مصاحبه نیمه ساختاریافته بود.

    یافته ها

    داده ها به روش شش مرحله ای تحلیل مضمون کلارک و براون تحلیل شدند. نتایج حاصل از تحلیل مصاحبه ها نشان داد که مدیریت توجه طلبی از سه مقوله ی اصلی ارزیابی (با سه مقوله ی سازمان دهنده ی ارزیابی خودگزارشی، ارزیابی علایم به کمک مصاحبه و ارزیابی آثار فردی و زوجی توجه طلبی)؛ درمان فردی (با مقوله های سازمان دهنده ی بینش یابی، اصلاح شناخت های محرک توجه طلبی، اصلاح رفتارها و هیجانات توجه طلبانه) و درمان زوجی با مقولات سازمان دهنده ی (بهبود دلبستگی، بهبود تمایز خود و بهبود قصه ی زندگی) تشکیل شده است.

    نتیجه گیری

    با توجه به نتایج، یک مدل درمانی سه بعدی از ارزیابی، درمان فردی و زوجی تشکیل شد که می تواند راهگشا و الهام بخش مشاوران علاقه مند به این حوزه باشد

    کلیدواژگان: تحلیل مضمون، توجه طلبی، خانواده، زنان متاهل، مدل درمانی
  • پندار جاهدعطائیان، حسن شمس اسفندآباد*، عبدالحسن فرهنگی صفحات 356-369
    هدف

    هدف پژوهش حاضر، اثربخشی روان شناسی مثبت بر خودشفقتی و قدردانی در بین زوجین دارای تعارضات زناشویی بود.

    روش پژوهش:

     این پژوهش از لحاظ هدف، کاربردی و از نظر گردآوری داده ها، نیمه آزمایشی با طرح پیش آزمون- پس آزمون با گروه گواه بود. جامعه آماری پژوهش، شامل زوجین دارای مشکلات زناشویی شهر تهران که در سال 1400 به مراکز مشاوره تهران (کلینیک تارا شعبه 1 و 2 و کلینیک بهجو) مراجعه کرده بودند (184 زوج) بودند که از میان آنها 30 زوج با شرط دارا بودن شرایط پژوهش و تمایل به شرکت در پژوهش به روش نمونه گیری هدفمند، انتخاب و به صورت تصادفی در یک گروه آزمایش و یک گروه گواه و در هر گروه 15 زوج، گمارده شدند. برای مداخلات درمانی روان شناسی مثبت از پروتکل رشید و سلیگمن شامل 10جلسه 90 دقیقه ای (2013) استفاده شد. ابزار پژوهش شامل  پرسشنامه پرسشنامه خودشفتقی نف (2003) و پرسشنامه قدردانی مک کلایف (2002) بود. داده ها با استفاده از آزمون تحلیل واریانس با اندازه گیری مکرر با استفاده از نرم افزار  SPSS- V26 تحلیل شد.

    یافته ها

    یافته ها نشان داد که در مرحله پس آزمون، تفاوت معنا داری بین گروه آزمایش با گروه گواه وجود دارد (001/0>P) و روش مداخله ای می تواند بر خودشفقتی (14/32 = F، 001/0 > P) و قدردانی زوجین (64/7 = F، 001/0 > P) درگیر، اثربخش واقع گردند.

    نتیجه گیری

    با توجه به نتایج پژوهش می توان بیان کرد که روان شناسی مثبت به عنوان یک روش درمانی موثر می تواند در بهبود، خودشفقتی و قدردانی و به تبع آن کاهش مشکلات زناشویی زوجین مورد استفاده قرار گیرد و به روانشناسان و متخصصان زوج درمانی توصیه می شود از این روش درمانی برای بهبود روابط بین زوجین استفاده کنند

    کلیدواژگان: جذابیت، خودشفقتی، قدردانی، روانشناسی مثبت
  • زهرا اسمعیلیان، محمدعلی رحمانی*، طاهر تیزدست صفحات 370-382
    هدف

    مطالعه حاضر با هدف تعیین میزان اثربخشی روان درمانی مثبت نگر بر تبعیت از درمان و هموگلوبین گلیکوزیله در بیماران زن مبتلا به دیابت نوع دو بود.

    روش پژوهش:

     این مطالعه از نوع نیمه تجربی باطر پیش آزمون، پس آزمون و پیگیری بود. 30 نفر از زنان مبتلا به دیابت نوع دو که از دی ماه تا اردیبهشت 1399 مراجعه کننده به انجمن دیابت ایران واقع در استان تهران با روش نمونه گیری هدفمند انتخاب شدند. سپس به صورت تصادفی در گروه آزمایش (تحت روان درمانی مثبت نگر) (n=15) و گروه گواه (n=15) گمارده شدند. ابزار جمع آوری اطلاعات مقیاس تبعیت از درمان (موریسکی، انگ و وود، 2008) و سنجش هموگلوبین گلیکوزیله در سه مرحله خط پایه، پس از مداخله و پیگیری سه ماهه جمع آوری شدند. ساختار جلسات روان درمانی مثبت نگر بر اساس مداخله گروهی روان درمانی مثبت نگر سلیگمن (2006) در هشت جلسه برای گروه آزمایش اجرا شد. تجزیه و تحلیل داده ها با استفاده از روش تحلیل واریانس اندازه گیری مکرر چند متغیری انجام شد.

    یافته ها

    یافته ها نشان داد که در گروه تحت روان درمانی مثبت نگر، میانگین نمره تبعیت از درمان به طرز معناداری بالاتر از گروه گواه است (16/511=F، 001/0>P). به علاوه، زنان مبتلا به دیابت گروه آزمایش در میانگین نمره هموگلوبین گلیکوزیله در مقایسه با گروه گواه به نحو قابل توجهی کاهش پیدا کردند (50/389=F، 001/0>P).

    نتیجه گیری

    نتایج این پژوهش نشان داد که روان درمانی مثبت نگر می تواند در بهبود تبعیت از درمان و کاهش هموگلوبین گلیکوزیله در زنان مبتلا به دیابت نوع دو موثر باشد.

    کلیدواژگان: تبعیت از درمان، دیابت نوع دو، روان درمانی مثبت نگر، هموگلوبین گلیکوزیله
  • لیلا اسدی، محمدرضا صیرفی*، مروارید احدی صفحات 383-400
    هدف

    این پژوهش با هدف تعیین اثربخشی نقاشی درمانی بر اضطراب سلامت، احساس تنهایی و اضطراب مرگ سالمندان انجام شد.

    روش پژوهش: 

    این پژوهش نیمه آزمایشی با طرح پیش آزمون- پس آزمون با گروه آزمایش و گروه کنترل با یک دوره آزمون پیگیری بود. از جامعه آماری سالمندان مقیم خانه های سالمندان شهرستان تهران در سال 1400 30 نمونه به صورت در دسترس انتخاب و به صورت تصادفی در دو گروه آزمایشی وگروه کنترل جایگزین شدند. نقاشی درمانی به مدت ده جلسه برای گروه آزمایش اعمال شد. یافته-های مورد نیاز با استفاده از پرسشنامه اضطراب سلامتی (سالکوسیس و وارویک، 2002)، احساس تنهایی (دهشیری و همکاران، 1386) و اضطراب مرگ تمپلر (1970) جمع آوری شد و با استفاده از تحلیل واریانس با اندازه گیری مکرر مورد تحلیل قرار گرفت.

    یافته ها

    نتایج پژوهش نشان داد که میانگین نمرات پس آزمون و آزمون پیگیری اضطراب سلامت، احساس تنهایی و اضطراب مرگ در گروه آزمایش کاهش پیدا کرد و نتایج تحلیل واریانس اندازه گیری مکرر نشان داد که مداخله نقاشی درمانی بر کاهش اضطراب سلامت، احساس تنهایی و اضطراب مرگ سالمندان اثربخش است (001/0 p<).

    نتیجه گیری

    بنابراین پیشنهاد می شود از مداخله نقاشی درمانی به منظور کاهش اضطراب سلامت، احساس تنهایی و اضطراب مرگ سالمندان استفاده شود.

    کلیدواژگان: سالمندان، اضطراب سلامت، احساس تنهایی، اضطراب مرگ، نقاشی درمانی
  • سارا مکی پور، عبدالله شفیع آبادی*، محمدرضا فلسفی نژاد، آناهیتا خدابخشی کولایی صفحات 401-425
    هدف

    هدف از پژوهش حاضر اثربخشی مدل غنی سازی خانواده برای کار بر رضایت زناشویی، رضایت شغلی و تعارض کار-خانواده مدیران زن متاهل بود.

    روش پژوهش: 

    مطالعه حاضر نیمه آزمایشی با گروه آزمایش و کنترل و طرح پیش آزمون- پس آزمون با پیگیری شش ماهه بود. جامعه آماری شامل کلیه 150 نفر مدیران زن متاهل شاغل در بخش ستادی بانک پارسیان شهر تهران در سال 1400-1399بودند که به همراه همسرانشان تمایل به شرکت در جلسات آموزشی داشتند.تعداد اعضای نمونه 30 زوج بود که با استفاده از روش نمونه گیری تصادفی در دو گروه آزمایش و کنترل جایگزین شدند(15زوج در گروه آزمایش و 15زوج در گروه گواه). گروه آزمایش به مدت 9 جلسه 90 دقیقه ای تحت مداخله مدل غنی سازی خانواده برای کار قرار گرفت.هردو گروه در سه مرحله پیش آزمون،پس آزمون و پیگیری به وسیله پرسشنامه های رضایت زناشویی انریچ مقیاس رضایت شغلی مینوسوتا (MSQ) و مقیاس تعارض کار-خانواده رستگارخالد (1381) مورد ارزیابی قرار گرفتند. داده ها با بهره گیری از تحلیل واریانس مختلط اندازه گیری مکرر تحلیل شدند.

    یافته ها

    نتایج حاکی از تاثیر مداخله غنی سازی خانواده برای کاربر نمره رضایت زناشویی (05/0 > p و 74/60= F) و رضایت شغلی (05/0 > p و 72/69= F) در مدیران زن و همچنین نمره تعارض کار-خانواده (05/0 > p و 17/8= F)، رضایت زناشویی (05/0 > p و 53/18= F) و رضایت شغلی (05/0 > p و 72/69= F) در زوجین و عدم تاثیر مداخله غنی سازی خانواده برای کاربر نمره تعارض کار-خانواده (05/0 < p و 95/0= F) در مدیران زن و پایداری این نتایج در مرحله پیگیری بود.

    نتیجه گیری

    با عنایت به نتایج به دست آمده از پژوهش، مدیران سازمانی می توانند از مدل آموزشی غنی سازی خانواده برای کار به عنوان یک روش مداخله ای اثربخش جهت بهبود رضایت زناشویی و شغلی مدیران زن متاهل و زوجین و کاهش تعارض کار-خانواده زوجین هر دوشاغل بهره مند شوند.

    کلیدواژگان: مدل غنی سازی خانواده برای کار، رضایت زناشویی، رضایت شغلی، تعارض کار-خانواده، زنان
  • پریا مژگانی، امید مرادی*، حمزه احمدیان، یحیی یاراحمدی صفحات 426-443
    هدف

    پژوهش حاضر با هدف مقایسه اثربخشی زوج درمانی به شیوه سایکودرام و زوج درمانی رفتاری تلفیقی بر بهبود تاکتیک های حل تعارض زوجین متعارض در شهر تهران 1398 انجام شد.

    روش پژوهش: 

    روش این پژوهش نیمه آزمایشی از نوع پیش آزمون-پس آزمون با گروه گواه و مرحله پیگیری است. جامعه آماری شامل کلیه زوجین متعارض مراجعه کننده به مرکز نمایش درمانی ایران در سال 1398 بود. نمونه این پژوهش شامل 36 نفر (18 زوج) از مراجعه کنندگان به مرکز نمایش درمانی ایران بود که به روش نمونه گیری در دسترس انتخاب شدند. سپس در سه گروه 12 نفری شامل دو گروه آزمایش و یک گروه گواه گمارش شدند. گروه آزمایش اول 12 جلسه تحت زوج درمانی به شیوه سایکودرام مورنو (1969) و گروه آزمایش دوم نیز 12 جلسه تحت زوج درمانی رفتاری تلفیقی کریستنسن و همکاران (2014) قرار گرفتند. داده ها با استفاده از مقیاس تجدید نظر شده تاکتیک های حل تعارض استراوس (1996) به دست آمد و با آزمون تحلیل واریانس چند متغیره و اندازه گیری مکرر در نرم افزار spss22 تحلیل شد.

    یافته ها

    نتایج این مطالعه نشان داد که زوج درمانی به شیوه سایکودرام بر حمله جسمی (52/11F= و 001/0P=) و خشونت روانی (74/12F= و 001/0P=) و مذاکره غیر خشن (84/58F= و 001/0P=) در فرم قربانی و به ترتیب حمله جسمی (96/17F= و 001/0P=)، خشونت روانی (08/17F= و 001/0P=) و مذاکره غیر خشن (45/89F= و 001/0P=) در فرم پرخاشگر موثر بوده است.

    نتیجه گیری

    نتایج حاکی از اثربخشی زوج درمانی به شیوه سایکودرام بر کاهش حمله جسمی و خشونت روانی و افزایش مذاکره غیر خشن در زوجین متعارض و ماندگاری آن در طی زمان بود. بنابراین متخصصان سلامت و درمانگران می توانند برای بهبود تاکتیک های حل تعارض بین زوجین از روش زوج درمانی به شیوه سایکودرام در کنار سایر روش های درمانی استفاده کنند.

    کلیدواژگان: تاکتیک های حل تعارض، زوج درمانی، سایکودرام، رفتاری تلفیقی
  • مرجان صادق خانی، سوزان امامی پور*، مالک میرهاشمی صفحات 444-471
    هدف

    پژوهش حاضر به منظور مدل یابی معادلات ساختاری طلاق عاطفی بر اساس صفات تاریک شخصیت و سبک های حل تعارض با میانجی گری دلزدگی زناشویی انجام شد.

    روش پژوهش: 

    روش پژوهش توصیفی از نوع همبستگی مبتنی بر معادلات ساختاری بود. جامعه آماری این پژوهش را کلیه زنان متاهل رده سنی 25-50 سال دانشگاه آزاد تهران واحدهای یادگار امام (ره)، ابوالفتوح رازی (مادران) و تهران شمال در نیمه نخست سال 1400 تشکیل دادند که از طریق روش نمونه-گیری در دسترس به صورت آنلاین (بدلیل شیوع ویروس کرونا) انتخاب شدند. ابزارهای پژوهش پرسشنامه های طلاق عاطفی گاتمن (EDQ،1994)، مثلث تاریک شخصیت (جانسون و وبستر، 2010)، سبک های حل تعارض رحیم (ROCI-II، 1983) و دلزدگی زناشویی پاینز(CBM،1996) بود. تحلیل ها با استفاده از نرم افزارهای SPSS26 و AMOS24 صورت گرفت.

    یافته ها

    نتایج حاصل از مدل سازی معادلات ساختاری نشان داد مدل مفروض پژوهش با داده های تجربی برازش دارد(063/0=RMSEA). بین صفت ماکیاولی (149/0= β)، سبک سازنده (441/0-=β) و سبک غیرسازنده (264/0=β) با طلاق عاطفی رابطه معناداری مشاهده شد. ضریب مسیر غیرمستقیم دلزدگی زناشویی در ارتباط بین صفت خودشیفته (007/0 = sig) و سبک های سازنده حل تعارض(043/0 =sig) با متغیر وابسته طلاق عاطفی معنادار بود اما نقش میانجی دلزدگی زناشویی در ارتباط بین سبک های غیر سازنده با طلاق عاطفی تایید نشد (536/0 = sig).

    نتیجه گیری

    صفات تاریک شخصیت و سبک های حل تعارض سازنده علاوه بر اثر مستقیم بر طلاق عاطفی می توانند با اثرگذاری بر دلزدگی زناشویی به طور غیر مستقیم بر آن تاثیرگذار باشند.

    کلیدواژگان: دلزدگی زناشویی، سبک های حل تعارض، صفات تاریک شخصیت، طلاق عاطفی
  • زهره زاد حسن، کبری کاظمیان مقدم* صفحات 472-486
    هدف

    این مطالعه با هدف تعیین اثربخشی زوج درمانی هیجان مدار بر فراهیجان مثبت، صمیمیت زناشویی و توانمندی های شخصیت زنان سالمند انجام شد.

    روش پژوهش:

     روش این مطالعه نیمه آزمایشی با طرح پیش آزمون- پس‫آزمون با گروه گواه بود.‬‬‬‬‬‬‬‬‬‬‬‬‬‬‬‬‬‬‬‬‬‬‬‬‬‬‬‬ جامعه آماری پژوهش کلیه زنان سالمند مراجعه کننده به مراکز مشاوره در شهر بابل در سال 1400 بودند. حجم نمونه پژوهش شامل 30 زن سالمند (بالای 60 سال) بودند که با روش نمونه‫گیری دردسترس و با توجه به ملاک های ورود و خروج مطالعه انتخاب شدند. گروه آزمایش مداخله زوج درمانی را 12 جلسه 90 دقیقه ای هفتگی دریافت نمود.‬‬‬‬‬‬‬‬ پرسشنامه های مورد استفاده در این پژوهش شامل پرسشنامه فراهیجان مثبت میتمانسگروبر، بک، هافر و شوبلر (2009)، صمیمیت زناشویی تامپسون و والکر (1993) و توانمندی های شخصیت پیترسون و سیلگمن (2004) بود. داده ها با تحلیل واریانس با اندازه گیری مکرر در نرم افزار spss نسخه 22 مورد تجزیه و تحلیل قرار گرفتند.

    یافته ها

    نتایج نشان داد که در پس آزمون بین گروه های آزمایش و کنترل از نظر فراهیجان مثبت، صمیمیت زناشویی و توانمندی های شخصیت تفاوت معنی داری وجود داشت (001/0>p). به عبارت دیگر، مداخله زوج درمانی هیجان مدار باعث بهبود میزان فراهیجان مثبت، صمیمیت زناشویی و توانمندی های شخصیت زنان سالمند شد (001/0>p).

    نتیجه گیری

    منطبق با یافته های پژوهش حاضر می توان مداخله زوج درمانی هیجان مدار را به عنوان یک روش کارا جهت افزایش فراهیجان مثبت، صمیمیت زناشویی و توانمندی های شخصیت زنان سالمند پیشنهاد داد.‬‬‬‬

    کلیدواژگان: زوج درمانی هیجان مدار، فراهیجان مثبت، صمیمیت زناشویی، توانمندی های شخصیت، زنان سالمند
  • پریسا خطیبی، پریسا مقراضی* صفحات 487-506
    هدف

    این پژوهش با هدف پیش بینی درماندگی روان شناختی بر اساس طرحواره ناسازگار اولیه و راهبردهای مقابله ای در زنان آسیب دیده از خیانت زناشویی انجام شد.

    روش پژوهش:

     پژوهش حاضر به لحاظ روش توصیفی، از نوع همبستگی است. جامعه آماری شامل کلیه کلیه زنان درگیر با مساله خیانت زناشویی همسر (مرد) مراجعه کننده به مراکز مشاوره منطقه 5 شهر تهران در سال 1400 بودند. به روش نمونه گیری در دسترس 210 نفر انتخاب شدند. ابزارگردآوری داده ها شامل پرسشنامه خیانت زناشویی ینیسری و کوکدمیر (2006)،  پرسشنامه درماندگی روان شناختی لویبوند و لویبوند (1995)، پرسشنامه راهبرد‎های مقابله‎ای لازاروس فولکمن (2001) و پرسشنامه طرح واره ناسازگار اولیه یانگ (2000) بود. تجزیه و تحلیل آماری با کمک ضریب همبستگی پیرسون و روش رگرسیون چندگانه انجام شد.

    یافته ها

    نتایج حاصل از تحقیق نشان داد که براساس طرحواره ناسازگار اولیه و راهبردهای مقابله ای می توان درماندگی روان شناختی را درر زنان آسیب دیده از خیانت زناشویی پیش بینی نمود.

    نتیجه گیری

    بررسی این متغیرهای پیش بین در مشاوره های پیش از ازدواج و خانواده برای شناسایی افراد در معرض خطر لازم است.

    کلیدواژگان: درماندگی روان شناختی، طرحواره ناسازگار اولیه، راهبردهای مقابله ای
  • مهدی احمدی، سعید اسلامی *، علی محمد مزیدی شرف آبادی صفحات 507-529
    هدف

    هدف از پژوهش حاضر، تدوین مدل کارکردهای شبکه های اجتماعی در کنترل بحران امنیت روانی خانواده (مرحله پس از بحران مطالعه موردی سیل خوزستان) بود.

    روش پژوهش: 

    این پژوهش در دو بخش کیفی و کمی انجام شد. در بخش کیفی، توسط مصاحبه و روش گراندد تیوری، مولفه های مدل کارکردهای شبکه های اجتماعی موثر بر کنترل بحران امنیت روانی خانواده در مرحله پس از بحران در 6 شبکه داده بنیاد شامل پدیده محوری، شرایط علی، شرایط زمینه ای، شرایط مداخله گر، راهبردها و پیامدها دسته بندی شدند. جامعه آماری در بخش کیفی، خبرگان شامل اساتید دانشگاهی متخصص در زمینه مدیریت بحران و کارکردهای شبکه های اجتماعی و همچنین خبرگان تجربی در زمینه مدیریت بحران سیل استان خوزستان که مشغول به فعالیت در این زمینه می باشند (مانند مدیران سازمان مدیریت بحران) هستند. روش نمونه گیری در بخش کیفی، از نوع سطح اشباع و هدفمند برابر 20 نفر بدست آمد. در بخش کمی، توسط پرسشنامه و روش معادلات ساختاری، مدل کیفی ارایه شده در بخش قبل آزمون گردید. جامعه آماری در بخش کمی، کارشناسان و کارکنان سازمان مدیریت بحران در ایران می باشند. نمونه گیری در بخش کمی به روش نمونه گیری تصادفی ساده برابر 384 نفر بدست آمد.

    یافته ها

    بر اساس نتایج بخش کیفی، کدهای شناسایی شده، در 6 شبکه اصلی، 16 مولفه اصلی و 87 مولفه فرعی دسته بندی شدند. بر اساس نتایج بخش کمی، شرایط علی با بار عاملی 588/0 بر پدیده محوری تاثیرگذار است. پدیده محوری، شرایط زمینه ای و شرایط مداخله گر به ترتیب با بارهای عاملی 515/0، 309/0 و 419/0 بر راهبردها تاثیرگذار هستند و راهبردها با بار عاملی 558/0 بر پیامدها موثر است.

    کلیدواژگان: کارکردهای شبکه های اجتماعی، کنترل بحران، امنیت روانی خانواده
  • سیده سارا ابوالحسنی، شهره قربان شیرودی *، محمدرضا زربخش بحری صفحات 561-564
    هدف

    این پژوهش با هدف تعیین اثربخشی زوج درمانی سیستمی تلفیقی بر تمایزیافتگی و دلزدگی زناشویی زوجین آسیب دیده از روابط فرازناشویی انجام شد.

    روش پژوهش: 

    طرح پژوهش شبه آزمایشی از نوع پیش آزمون و پس آزمون و پیگیری با گروه گواه بود. جامعه آماری زنان آسیب دیده از روابط فرازناشویی مراجعه کننده به مراکز مشاوره شهر بابل در نه ماهه اول سال 1398 بودند. تعداد 30 نفر با روش نمونه گیری دردسترس انتخاب و به صورت تصادفی در دو گروه 15 نفره آزمایش و گواه قرار گرفتند. سپس افراد گروه آزمایش پروتکل زوج درمانی سیستمی تلفیقی فلدمن (1990) را در 12 جلسه 90 دقیقه ای به صورت گروهی تحت آموزش قرار گرفتند و گروه گواه نیز آموزشی دریافت نکرد. برای جمع آوری اطلاعات از پرسشنامه های دلزدگی زناشویی پاینز (1996) و پرسشنامه تمایز یافتگی خود اسکورون و فرید لندر (1989) استفاده شد. داده ها با استفاده از آزمون اندازه گیری مکرر تحلیل شد.

    یافته ها

    نتایج نشان داد که زوج درمانی سیستمی تلفیقی بر تمایزیافتگی (01/30=F، 001/0>P) و دلزدگی زناشویی (86/78=F، 001/0>P) زنان آسیب دیده از روابط فرازناشویی در مرحله پس آزمون تاثیر معنادار داشت. همچنین اثربخشی زوج درمانی سیستمی تلفیقی بر تمایزیافتگی و دلزدگی زناشویی در مرحله پیگیری نیز تداوم داشت (05/0P<).

    نتیجه گیری

    با توجه به یافته‎ها این پژوهش می توان از طریق زوج درمانی سیستمی تلفیقی به شیوه گروهی به بهبود تمایزیافتگی و دلزدگی زناشویی در زنان آسیب دیده از روابط فرازناشویی کمک نمود.

    کلیدواژگان: زوج درمانی سیستمی تلفیقی، تمایزیافتگی، دلزدگی زناشویی، روابط فرازناشویی
  • فاطمه آزادسروستانی *، محبوبه مینا صفحات 630-645
    هدف

     هدف از پژوهش حاضر بازنگری در برخی از مواد قانون حمایت خانواده مصوب 1391 و قانون مدنی است تا بتواند از فروپاشی بنیان خانواده درمواردی که موضوع آن عیوب (اختلالات) جنسی زوجین است جلوگیری کند. 

    روش پژوهش: 

    پژوهش حاضر از روش توصیفی - تحلیلی بهره برده و با استفاده از آرای قضایی، کتب، پایان نامه و مقالات معتبر به رشته تحریر درآمده است. 

    یافته ها

     براساس یافته های این پژوهش در راستای استحکام بنیان خانواده باید اختلالات جنسی زوجین قبل از ثبت نکاح توسط متخصصین بررسی گردد و در انحلال نکاح به سبب فسخ و طلاق نیز دادگاه باید در جریان رسیدگی با هدف اصلاح بین زوجین آن ها را الزام به درمان نماید. به منظور حفظ حقوق زوجه در شرایط خاصی با اینکه عیب زوج قابل درمان است اما هم چنان حق فسخ نکاح برای زوجه به استناد عیب باقی است و با تصریح اختلالات جنسی در تبصره ماده 1130 قانون مدنی دشواری اثبات عسروحرج از زوجه ساقط می شود. 

    نتیجه گیری

     نباید اختلالات جنسی که قابل درمان هستند را در تمام موارد موجب انحلال نکاح دانست و دادگاه باید زوجین را برای درمان به مراکز تخصصی معرفی کند. بازنگری در برخی از مواد قانون مدنی و قانون حمایت خانواده مصوب 1391 می تواند موجب تشخیص و درمان به موقع اختلالات جنسی زوجین گردد و آمار انحلال نکاح را کاهش دهد.

    کلیدواژگان: طلاق به درخواست زوجه، طلاق درحقوق کشورهای اسلامی، عیوب (اختلالات) جنسی، فسخ نکاح
|
  • Zohreh Ansari, Fariba Naderian, Fatemeh Abedini, Maryam Yavari Kermani * Pages 1-28
    Aim

     The aim of this study was to determine the effectiveness of emotion-oriented couple therapy and compare it with structural couple therapy on marital justice and relationship maintenance strategies in conflicting couples. 

    Method

     The present study was a quasi-experimental with pre-test and post-test with a two-month control and follow-up group; The statistical population of the present study was 205 couples referring to counseling centers in Shiraz in the last six months of 1399. After screening with the Sanai Marital Conflict Questionnaire (2008), 60 couples who received more than 150 points from the cut-off point were selected by available means. They answered the Marital Justice Questionnaire and Relationship Strategies (Stafford, Dinton and Haas, 2000) and in the experimental group of emotion-focused couple therapy (10 couples), the experimental group of structural couple therapy (10 couples) and the control group (10 couples) were randomly replaced. Both experimental groups underwent Johnson (2004) emotion-focused couple therapy and Minochin (1974) structural couple therapy intervention for eight 90-minute sessions, and the control group did not receive any intervention. Data were analyzed using repeated measures mixed variance analysis. 

    Results

     The results showed that emotion-oriented couple therapy (F = 8.97 and P = 0.005) and structural couple therapy (F = 10.04 and P = 0.003) on increasing marital justice and emotion-oriented couple therapy (0.60). F = 59 and P = 0.001) and structural couple therapy (F = 19.29 and P = 0.001) are effective in maintaining relationship strategies in conflicting couples and this effect was stable in the follow-up phase; The results of post hoc test also showed that the difference between the two groups of emotion-oriented couple therapy and structural couple therapy in marital justice variables and strategies for maintaining the relationship is not significant. 

    Conclusion

     The results of the research suggest evidence that the intervention of emotion-oriented couple therapy and structural couple therapy is a suitable method for increasing marital justice and relationship maintenance strategies in spouses suffering from marital conflict.

    Keywords: Marital conflict, emotion-oriented, Structural, marital justice, relationship maintenance strategies
  • Mahshid Niknezhadi, Ilnaz Sajjadian *, Gholamreza Manshaee Pages 29-46
    Aim

    This study aimed to determine the effectiveness of emotion efficacy therapy on self-control and intimacy of women injured by trauma husband infidelity.

    Methods

    The research method was quasi-experimental and pretest-posttest design with two-months follow-up. The statistical population was the women injured by trauma husband infidelity who referred to counseling centers in Isfahan city in 2019, after screening with the post-traumatic stress disorder measurement scale (Wedders et al., 1994), 30 people were selected by purposive sampling and they were randomly replaced in two experimental and control groups (15 people in each group). The research tools were self-control questionnaire (Tangeny et al., 2004) and intimacy (Walker & Thompson, 1983). The data were analyzed using repeated measures analysis of variance. The experimental group underwent 8 sessions of 90-minute emotion efficacy therapy intervention by Matthew et al. (2016) but the control group received no treatment.

    Results

    The results showed the effect of emotion efficacy therapy on self-control (P<0/001, F=22/479) and intimacy (P<0/001, F=57/14) in post-test, and stability of this effect was in the follow-up.

    Conclusion

    According to the results of this research, emotional efficiency therapy can be used to improve the self-control and intimacy of women Injured by trauma husband’s infidelity.

    Keywords: Emotion Efficacy Therapy, self-control, intimacy, Trauma, Husband Infidelity
  • Mohamadreza Mehdigholi, Farideh Dokaneifard *, Pantea Jahangir Pages 47-59
    Aim

    Due to the lack of experience and resilience in marital conflicts, young couples are more exposed to marital heartbreak and as a result tend to divorce. This research was conducted with the aim of determining the effectiveness of emotion-focused couple therapy on marital burnout and resilience of young couples.

    Methods

    A quasi-experimental research method was used with a pre-test-post-test design with a control group. 30 couples (30 men and 30 women) were selected from the statistical population of young couples who referred to family counseling centers in the 1st district of Tehran using the available sampling method and were replaced in two experimental and control groups using a random method. The research tools were marital burnout (Pines, 1996), resilience (Connor & Davidson, 2003). The research findings were analyzed using SPSS version 23 software and analysis of variance with repeated measurements.

    Results

    The results showed that emotional couple therapy has a significant and stable effect on reducing marital boredom and increasing resilience of young couples (p<0.001).

    Conclusion

    Therefore, in order to improve marital burnout and resilience of young couples, it is suggested to use the emotion-oriented couple therapy approach

    Keywords: Emotion-focused Couple Therapy, Marital Burnout, Resilience, young couples
  • Fatemeh Malma Ghobadi, Elham Salehi Rizi *, Abdolreza Afzood Pages 60-78
    Aim

    The purpose of this research is to investigate the structural model of marital commitment based on irrational beliefs and marital justice with the mediation of cognitive emotion regulation.

    Methods

    The research method was correlation and structural equation modeling. The statistical population in this research included all couples who referred to Tehran counseling centers in 2021. Purposive sampling method was used to select the research sample, so that 300 couples were selected as the sample group based on the criteria for entering and exiting the research. The research tools included the Jones Irrational Beliefs Questionnaire (1968), the Marital Commitment Questionnaire of Adams and Jones (1997), the Cognitive Emotion Regulation Questionnaire of Garnefski and Kraaij (2006), and the Marital Justice Scale of Ghaffari et al. (2018). Structural equation modeling method was used for data analysis.

    Results

    The results of the conceptual model test showed that the fit indices of the research model are in a favorable condition. Irrational beliefs have a significant indirect effect (through positive emotion regulation and negative emotion regulation) on couples' marital commitment (p<0.05). Marital justice and self-efficacy affect patients' marital commitment through negative emotion regulation (p<0.05).

    Conclusion

    According to research findings, in counseling centers, to increase the level of marital commitment, it is necessary to pay attention to the antecedents of irrational beliefs and marital justice

    Keywords: Marital Commitment, Irrational Beliefs, marital justice, Cognitive emotion regulation
  • Zainab Kianoosh, Bahram Mirzaian *, Hamid Nejat Pages 79-95
    Aim

    The purpose of this study was to compare the effectiveness of cognitive-behavioral therapy and parent-child interaction therapy on the anxiety and loneliness of children with separation anxiety disorder.

    Methods

    The current research was of the type of practical and quasi-experimental designs of pre-test-post-test and follow-up with a control group. The statistical population of this research included all children (girls and boys) suffering from separation anxiety disorder in Sari city in the first six months of 2021. The sample of this research includes 30 people who referred to Bammad Counseling Center and Bawar Counseling Center who were selected using available sampling method and randomly divided into two experimental groups of cognitive-behavioral therapy and parent-child interaction therapy and the control group in the list. They waited. The data were obtained using the Spence Children's Anxiety Scale (1997) and Asher et al.'s (1984) Loneliness Questionnaire. The data was analyzed using the method of analysis of variance with repeated measurements and SPSS-26 software.

    Results

    The results showed that there is a significant difference between the two groups of cognitive-behavioral therapy and parent-child interaction therapy on anxiety and loneliness in children with separation anxiety disorder, and cognitive-behavioral therapy causes more changes in anxiety in children. He was diagnosed with separation anxiety disorder and this treatment is stronger than parent-child interaction therapy in this group of patients. However, according to the average indices and the effect size obtained in the treatment of parent-child interaction, it caused more changes in loneliness in children with separation anxiety disorder, and this treatment is stronger than cognitive-behavioral therapy in this group of children with separation anxiety disorder. It was separation anxiety disorder (P<0.001).

    Conclusion

    Therefore, it can be concluded that cognitive-behavioral treatment of anxiety in children with separation anxiety disorder and parent-child interaction therapy can be an effective intervention method to improve the loneliness of children with separation anxiety disorder.

    Keywords: cognitive-behavioral therapy, parent-child interaction therapy, anxiety, Loneliness, Separation anxiety disorder
  • Azam Sarabadani, Ramezan Hasanzadeh *, Hossein Ali Ghanadzadegan Pages 96-114
    Aim

    The present study aims to investigate the effectiveness of treatment based on acceptance and commitment on distress tolerance and cognitive regulation of emotions in women with generalized anxiety disorder.

    Method

    It was a semi-experimental type with a pre-test-post-test design with a control group (one experimental group and one control group) and a three-month follow-up. The statistical population of women with generalized anxiety disorder referred to counseling centers and health homes in districts 1, 2, 3 and 5 of Tehran in the first half of 2021. The sample consisted of 30 women with generalized anxiety disorder who were selected in a purposeful and accessible manner with the generalized anxiety test and having the entry criteria. to collect data from tools; "Distress Tolerance Scale", "Cognitive Emotion Regulation Questionnaire" and "Pervasive Anxiety Scale" were used. The experimental group received acceptance and commitment-based treatment intervention during two months in 8 90-minute sessions. Meanwhile, the group Evidence of this intervention was not received during the research process and a follow-up period was conducted after three months. The collected data were analyzed in SPSS.27 software.

    Results

    The results showed that the treatment of acceptance and commitment was significantly tolerated. increased distress, adaptive strategies significantly increased cognitive regulation of emotion, and non-adaptive strategies significantly decreased cognitive regulation, and there was a significant difference in both variables between pre-test and post-test, as well as between pre-test and follow-up period.

    Conclusion

    Treatment based on acceptance and commitment affects women with generalized anxiety disorder and improves distress tolerance and cognitive regulation of emotion in them. It is recommended to use treatment based on acceptance and commitment for women with generalized anxiety disorder.

    Keywords: acceptance, commitment, Distress tolerance, Cognitive emotion regulation, generalized anxiety disorder
  • Shiva Vatanpanah, Javad Khalatbari *, Afshin Tayyebi, Mehrdad Sabet Pages 115-128
    Aim

    The present study was conducted with the aim of determining the effectiveness of compassion-focused therapy on emotional eating behavior, emotional dysregulation, perceived stress, and rumination in women with chronic obesity.

    Method

    The current research was part of the semi-experimental designs with pre-test-post-test-follow-up with control group. The statistical population of the current study included all women with chronic obesity and body mass index who referred to the Kian nutrition counseling clinic in Tehran between May and July 2022. In order to form two groups using the purposeful sampling method (taking into account the entry and exit criteria), first 30 women with chronic obesity and body mass index were selected by screening method and then randomly 15 patients in the experimental group and 15 patients in the group The witnesses were grouped. The program of compassion-focused treatment sessions during 8 90-minute sessions was formed for the experimental group based on the protocol of Neff and Germer (2013). Also, the follow-up phase was performed three months after the post-test on all three groups. The research tools included the emotional eating behavior questionnaire of van Strein et al. (2009), the emotional dysregulation questionnaire of Gratz and Romer (2004), the perceived stress questionnaire of Cohen et al. (1983) and the rumination questionnaire of Nolen Hoeksma and Marrow (1991). The data of the research were analyzed using repeated measurement analysis of variance test.

    Results

    The results of data analysis showed that the treatment focused on compassion in the participants of the experimental groups compared to the control group had a significant effect on improving emotional eating behavior (F=115.83, P<0.001), emotional dysregulation (F=98.10, P<0.001), perceived stress (F= 101.53, P<0.001) and rumination (F= 74.52, P< 0.001) in women with chronic obesity and body mass index.

    Conclusion

    Therefore, it can be concluded that compassion-focused therapy improves emotional eating behavior, emotional dysregulation, perceived stress, and rumination in women with chronic obesity.

    Keywords: compassion, emotional eating behavior, Emotional dysregulation, Perceived stress, Rumination
  • Hassanpour Amaneh, Ebrahim Alizadeh Mousavi *, Mohammad Mohammadipour Pages 129-142
    Aim

    The purpose of the research is to Investigating the mediating role of rumination between parental control and shyness in adolescent girls.

    Method

    The current research method is descriptive-survey of correlation type. The purpose of the research is applied and cross-sectional in terms of time. The statistical population includes all female secondary school students of Bojnord city who were engaged in the analysis in the academic year of 2014-2016. In order to estimate the sufficient sample size, Klein's (2005) sample size determination method was used, and 567 people were selected as the sample size. The sampling method is a single-stage cluster. In order to collect the desired information and measure the research variables, Hoeksma and Maro (1991) Rumination Questionnaire, Leslie et al.'s (1986) Parental Control (PLOC) and Stanford Shyness Questionnaire were used. SPSS and AMOS software were used to analyze the data.

    Results

    According to the results, the causal model of students' shyness based on parental control with the mediating role of rumination has a suitable fit.

    Conclusion

    parental control has a direct effect on self-criticism. Parental control has a direct effect on shyness. Parental control has a direct effect on rumination. Rumination has a direct effect on modesty. Rumination plays a mediating role between parental control and shyness.

    Keywords: Rumination, parental control, shyness, teenage girls
  • Alexis Gregorash, Eman Tadros *, Anha Jhuremalani Pages 143-155

    Incarceration affects approximately 2 million individuals, with about half the male population reporting they are in an intimate relationship; causing non-incarcerated partners to experience various stressors and challenges. Specific to women, there is a phenomenon called “Superwoman Syndrome,” which refers to various the roles women take on such as caregiver, partner, career woman, and breadwinner. This may lead to higher rates of stress and dissatisfaction, decreased family functioning, and dysfunctional patterns. This can also exacerbate feelings of resentment, dissatisfaction, and inundation, proliferating into negative feelings towards the family unit. Partner incarceration can exasperate these impacts, leading to higher rates of stress and other mental health concerns. Our goal is to address the lacunae in literature regarding the social, emotional, financial, and systemic needs of females with incarcerated partners, through advocacy of policy changes. Further, empirical exploration and research into this population can help to mitigate some of the negative effects, and produce better outcomes for both individual women, and their families.

    Keywords: incarceration, superwoman syndrome, Race, mental health
  • Mitra Mohammadian Khansari, Ali Asghar Hosseinzadeh * Pages 156-172
    Aim
    The aim of this study was to predict the attitude of single young people to marriage based on personality traits and perfectionism.
    Method
    The present research was descriptive-correlational in terms of applied purpose and descriptive research design. The statistical population included all single students of Isfahan universities in the academic year 2020-2021. From this community, 170 students were selected by available methods and all paticipants responded to Bratton and Rosen (1998) Attitudes Towards Marriage, the 5 Big Factors of Short Form Personality (NEO-FFI), and the Terry - Short et al. (1995) Perfectionism Questionnaires. Pearson correlation method and hierarchical regression analysis and SPSS software were used to analyze the data.
    Results
    The results showed that among personality traits, neuroticism negatively and extroversion positively and significantly predicts the attitude to marriage. Also, negative perfectionism negatively and positive perfectionism positively and significantly predict the attitudes towards marriage.
    Conclusion
    Based on the research findings, it can be concluded that personality traits and perfectionism are among the factors that play a role in young people's attitudes toward marriage.
    Keywords: Attitude towards marriage, personality traits, perfectionism
  • Mojtabah Reisi Sarteshneizy *, Ameneh Reisi Sarteshneizy Pages 173-185
    Aim
    The purpose of this study was to investigate the effect of emotion-oriented couple counseling on threatened self-image.
    Method
    The method of this research was semi-experimental and pre-test-post-test type with a control group. The statistical population of this research included couples who referred to the family culture center and counseling centers in Isfahan city. To select the desired sample, 40 couples (80 people) were selected as volunteers and randomly divided into two experimental and control groups (20 couples in the experimental group and 20 couples in the control group). The research tool was the threatened self-image from the NI-90 questionnaire. The independent variable was emotion-oriented couple counseling, which was applied to the experimental group in 10 sessions.
    Results
    The results of covariance analysis showed that emotion-oriented couple counseling had an effect on the threatened self-image, but it had no effect on the dimensions of hope, distortion of reality, departure to a feeling of harmony, and the threatened self variable.
    Conclusion
    it can be concluded that emotion-oriented couple counseling reduces the self-threatening image of couples
    Keywords: Emotion-Oriented Couple Counseling, Negative Emotions, Threatened Self-Image
  • Elham Khajenezami, Vida Sadat Razavi *, Vida Andishmand, Amanallah Soltani Pages 186-204
    Aim

    The aim of this study was to design and validate a stress and anger management training package based on book therapy and play therapy in primary school children in Kerman province:

    Method

    This research is multifaceted. Led to the choice of a qualitative-quantitative mixed research method). Among the mixed research methods, a sequential mixed exploratory research design will be used. In order to collect data, in addition to using library studies and theoretical foundations derived from content analysis, in-depth semi-structured interviews will be used for further understanding.

    Results

    The results of this study show that play therapy, story therapy packages can be used by psychologists to treat stress and anger in children. The results also showed that the mean of aggression and stress after using play therapy and book therapy, decreased significantly.

    Conclusion

    The findings of this study indicate that the use of these methods reduces children's stress and increases their acceptance in society.

    Keywords: educational package, stress, anger management, book therapy, Play therapy, children
  • Parisa Sabzevari, Gholamreza Khoyinejad *, Mohamadreza Saffarian Toosi Pages 205-226
    Aim

    Emotional divorce as a social harm is the first step in the divorce process and indicates the severity of marital conflicts originating from numerous factors such as the malfunctioning of couples' communication patterns. Hence, the present study aimed at comparing the effectiveness of Bowen's family-based approach and Minuchin’s structural family therapeutic approach on improving communication patterns and reducing marital conflicts in women suffering from emotional divorce.

    Method

    The quasi-experimental research method was based on pretest-posttest along with a control group. The population included all the women suffering from marital conflicts referring to the Central Welfare Organization of Isfahan in the first three months of the year 1399. After sifting through the subjects, 45 samples were selected by simple random sampling and were categorized into two experimental and control groups. The tools applied in the research were the scale of communication patterns and marital conflicts. The samples of the experimental group attended the Bowen's and Minuchin’s family therapeutic training programs, but the control group did not receive any particular training. Finally, the data were analyzed at two levels of descriptive statistics (the Mean and the Standard Deviation) and inferential statistics (the analysis of Covariance and the analysis of Variance with multivariable frequent measuring) using SPSS software.

    Results

    Findings revealed that both Bowen's and Minuchin’s therapeutic approaches can be effective on communication patterns and marital conflicts. In addition, a significant difference was noticed between the effectiveness of the two experimental groups (Bowen’s and Minuchin’s) on communication patterns of women suffering from emotional divorce, and the comparison of means suggested that family and structural therapy of Minuchin performed better. And finally, no significant difference was noticed between the effectiveness of the two experimental groups (Bowen’s and Minuchin’s) on marital conflicts in women suffering from emotional divorce, and only after lessening relationship with relatives, the structural effectiveness of Minuchin's approach proved to be better.

    Conclusions

    Therefore, Bowen’s and Minuchin’s therapeutic approaches can both be helpful in reducing the rate of marital conflicts and improving the communication patterns of women suffering from emotional divorce, and consequently, reducing emotional divorce and increasing marital adjustments.

    Keywords: Family Therapy, Communication Patterns, Marital conflicts, emotional divorce
  • Fatemeh Heidari, Hasan Ahadi *, Biuok Tajeri Pages 227-240
    Aim

    Infertile women face a lot of anxiety in life, and it seems that treatments derived from the third wave of psychotherapy can be effective in reducing their anxiety. As a result, the aim of this study was comparing the effectiveness of compassion focused therapy and mindfulness-based cognitive therapy on the anxiety of infertile women.

    Methods

    The present research in terms of purpose was applied and in terms of implementation method was semi-experimental with a pretest, posttest and two month follow-up plan with the control group. The research population consisted of infertile women who referred to clinics with a fertility specialist of Tehran city in 2022 year. The research sample was 45 people who were selected by purposive sampling method after checking the inclusion criteria and randomly divided into three equal groups including two experimental groups and one control group. Each of the experimental groups underwent eight sessions of 90-minute the therapies of compassion focused and mindfulness-based cognitive, separately and the control group during this time did not receive therapy. The data were collected with Beck et al's anxiety inventory (1988) and analyzed with mixed variance analysis and Bonferroni post hoc test in SPSS software.

    Results

    The analyzes showed that both compassion focused therapy and mindfulness-based cognitive therapy in compared to the control group reduced the anxiety of infertile women, and the effectiveness of compassion focused therapy in compared to mindfulness-based cognitive therapy was more effective in reducing their anxiety (P<0.001).

    Conclusion

    According to the results of the present study and considering the greater effectiveness of compassion focused therapy in compared to mindfulness-based cognitive therapy in reducing the anxiety of infertile women, it is suggested that therapists in their interventions used from compassion focused therapy.

    Keywords: compassion focused therapy, Mindfulness-based cognitive therapy, anxiety, Infertile Women
  • Sedigheh Afshari, Seyed Abdulmajid Bahrainian *, Fatemeh Shahabizadeh Pages 241-257
    Aim

    The purpose of this research was to determine the effectiveness of social-emotional skills training on motivational beliefs and self-regulation learning strategies of girls with learning disabilities.

    Methods

    The present research method is practical in terms of purpose and in terms of field situation and in terms of semi-experimental method with the design of pre-test, post-test and follow-up was with the control group and random assignment. The statistical population of this research included all girls aged 10 to 13 with learning disabilities in reading, writing and math in Joybar city in the academic year of 2019-20. Sampling was purposeful and 30 girls were selected as a sample, and then 15 girls were randomly assigned to the social-emotional skills training group and 15 girls were assigned to the control group. The experimental group was trained in social-emotional skills based on the Squires model (2003) for 10 sessions; while the control group did not receive any intervention. Also, Harter's motivational beliefs questionnaire (1981) and Ryan and Connell's self-regulated learning questionnaire (1989) were used in the pre-test, post-test and follow-up stages. Finally, the data were analyzed using ANOVA with repeated measures and Bonferroni's post hoc test.

    Results

    The results showed that social emotional skills training has an effect on girls' motivational beliefs (F=16.95; P=0.002) and self-regulation learning strategies (F=88.75; P=0.001).

    Conclusion

    It can be concluded that social emotional skills training is effective on girls' motivational beliefs and self-regulation learning strategies.

    Keywords: social emotional skills, Motivational Beliefs, self-regulated learning, Learning Disorder
  • Saeedeh Farajnia, Mohammad Reza Beliad *, Simin Hosseinian, Marjan Hosseinzadeh Taqvai Pages 258-278
    Aim

    One of the most important relationships between a man and a woman is marriage, which includes both emotional and legal commitment and is of special importance in the life of every adult. Therefore, the purpose of this research was to model marital adjustment based on communication patterns and marital relationship performance with the mediation of spiritual attitude.

    Method

    The present research was a descriptive-correlation type of structural equation modeling. The statistical population of the research included married people living in Tehran, according to the number of observable changes, 350 samples were available according to Spanir's (1976) marital compatibility questionnaires, spiritual attitude: Shahidi and Farajnia (2013), Christensen and Salavi's communication patterns. (1984) and sexual performance: Farajnia et al. (2013) answered. The research data was also analyzed through SPSS and AMOS software.

    Results

    The findings showed that the constructive communication pattern directly explains β = 0.334 of the variance of marital adjustment, and the withdrawal communication pattern directly explains β = 0.159 of the variance of marital adjustment. Also, the constructive communication pattern directly explains β=0.21 of the variance of spiritual attitude and the spiritual attitude directly explains β=0.147 of the variance of marital adjustment.

    Conclusion

    It can be concluded that the modeling of marital compatibility based on communication patterns and marital relationship performance with the mediation of spiritual attitude is valid.

    Keywords: marital adaptability, Communication Patterns, sexual relationship, Spiritual Attitude
  • Majid Mahmoodi Mozafar * Pages 279-290
    Aim

    Marriage can be stable or unstable according to various factors, including interpersonal problems of the couple. The aim of the research was to determine the effectiveness of the premarital counseling package training based on the integration of schema therapy and imagotherapy approaches on the treatment of interpersonal problems of couples on the verge of marriage.

    Research Method

    The research was a semi-experimental method with a pre-test, post-test and two-month follow-up plan with the control group. The statistical population consisted of boys and girls on the verge of marriage who referred to Rah Farda Counseling Center in Tehran in 2021, and twenty couples were selected by random sampling. Then they were randomly divided into two groups consisting of 10 test pairs and 10 control pairs. For both groups, pre-test, post-test and follow-up were performed. Interpersonal problems questionnaire (Hurwitz et al., 1989) was used. The counseling package was conducted in eight ninety-minute sessions twice a week only for the experimental group. Data analysis was done using repeated measures analysis of variance and LSD post hoc test in SPSS version 22 software. The significance level of statistical tests was considered 0.05.

    Results

    The results showed that the main effect of time (p = 0.006), the main effect of group (p = 0.001) and the interaction effect of time and group (p = 0.001) were significant on the variable of interpersonal problems. In the experimental group, the average pre-test scores of the couple's interpersonal problems variable were significantly higher than the average scores of the post-test (p = 0.015) and follow-up (p = 0.024). In addition, there was no significant difference between the mean scores of the mentioned variable between the post-test and follow-up stages (p = 0.228), which means that the effect of the intervention of trainings is lasting over time in the experimental group.

    Conclusion

    Based on the results, pre-marriage counseling package based on the integration of approaches and concepts of schema therapy and imagotherapy is effective in reducing the interpersonal problems of couples on the eve of marriage

    Keywords: Pre-marital counseling package, interpersonal problems, Couples on the verge of marriage
  • Morteza Haydari *, Zaynab Sadat Mirshabani, Atefeh Sadegh Masjedi, Hanieh Mortezaei Pages 291-303
    Aim

    The present study was conducted to investigate the effectiveness of cognitive-based mindfulness group therapy on reducing anxiety and increasing the quality of life of couples with delinquent husbands.

    Method

    The current research design was a quasi-experimental pre-test-post-test and follow-up with a control group. The statistical population included all criminals and their spouses who refer to Wasal Psychological and Counseling Clinic during the year 2021. For sampling, the available sampling method is used, 20 couples are selected from the clients of Vesal Psychological and Counseling Clinic and are randomly replaced in the experimental group and the control group. Received the cognitive-based intervention and no intervention was provided to the members of the control group. Before and after the implementation of the intervention, Beck's anxiety questionnaire (Beck et al., 1990) and quality of life (World Health Organization group, 1998) were completed by the sample group.

    Result

    The results of repeated measurement variance analysis showed that in the post-test and follow-up stages, the anxiety of women with delinquent husbands in the experimental group compared to the control group showed a significant decrease (F = 89.74, P = 0.001) and the quality of life of the experimental group compared to the control group. The evidence of increase (F = 110.31, P = 0.001) was found significant.

    Conclusion

    The findings of the research confirm that it is important that mindfulness group therapy based on reducing anxiety reduces the marital conflicts of couples with delinquent spouses

    Keywords: consciousness, anxiety, Quality of Life, women with delinquent husbands
  • Reihaneh Fakhrezare, Mojtaba Ansari Shahidi *, Neda Adibi Pages 304-314
    Aim

    The purpose of this study is to investigate the relationship between quality of life and acne disease.

    Methods

    In this study, a descriptive study method has used. There were 93 patients aged 15 to 50 years with acne skin disorder who included in the study by available sampling method. The GAGS rating scale was used for the severity of the disease. Also, The Skin Quality of Life Questionnaire (DLQI), was used to determine the quality of life of acne patients.

    Results

    Quality of life was not normal among the acne severity groups, so the Kruskal-Wallis test was used to calculate the correlation, which stated that there is a significant correlation between these two variables (P<0.001).

    Conclusion

    Quality of life and acne vulgaris have a strong correlation. Considering the frequency of this disorder and its effect on the social and emotional performance of people, it is necessary to investigate more in order to prevent and treat this disorder

    Keywords: Acne, Quality of Life, psychological effects
  • Mina Deldadeh Mehraban, Ramezan Hasanzadeh *, Bahram Mirzaian Pages 315-323
    Aim

    The aim of this study was to compare the effectiveness of schema-based couple therapy approach and act on marital boredom, emotion regulation in nurses with marital conflicts.

    Method

    The present research method is experimental. The research design is quasi-experimental with pre-test-post-test design and one-month follow-up with the control group. The statistical population included all nurses serving in Imam Sajjad Government Hospital in Ramsar in 2020. Among them, 30 couples (90 people) were selected by available non-random sampling method and were replaced in the experimental group (couple therapy based on schema and act) and a control group by simple random sampling (15 couples in each group). Then, 10 90-minute sessions (2 sessions per week) of schema and act-based couple therapy (Leo and McKee, 2017) were performed in experimental groups. Data were collected using the Sanai and Barati Marital Conflict Questionnaire (2000), the Pines Marital Boredom Questionnaire (1996) and the Gross and John Emotion Regulation Questionnaire (2003). Data were analyzed using SPSS-24 software and repeated measures analysis of variance.

    Results

    The results showed that schema-based couple therapy based on marital boredom (P<0.001, F=13.53) and emotion regulation (P<0.001, F=49.86) in nurses with Marital conflicts are effective.

    Conclusion

    Considering the effectiveness of couple therapy based on schema and act in nurses with marital conflicts, it is recommended that therapists use this therapeutic and educational approach

    Keywords: Schema therapy, Acceptance, Commitment Therapy, marital boredom, emotion regulation
  • Simin Soleymani, Lida Leilabadi *, Marjan Jafari Roshan Pages 324-339
    Aim

    The aim of this study was to compare the effectiveness of hope-based therapy and Acceptance and Commitment Therapy on depression and anxiety death of patients with type 2 diabetes

    Method

    This study was implemented using a pre-test and post-test quasi-experimental design with a control group. a total of 45 patients with type 2 diabetes referred to the Iranian Diabetes Association who met the inclusion criteria were selected by available sampling method and randomly assigned to the control and experimental. For the experimental group, hope-based treatment and Acceptance and commitment Therapy was performed but the control group did not receive any treatment. both groups completed the Depression Inventory (Beck et al., 1996) and the Death Anxiety Inventory (Templer, 1970) before and after the clinical intervention. Data were analyzed using mixed analysis of variance.

    Results

    The results showed that the effectiveness of hope-based therapy and acceptance and commitment therapy reduced depression and death anxiety in patients with type 2 diabetes. It was more effective than hope-based treatment in patients with type 2 diabetes.

    Conclusion

    The results of this study suggest that in addition to follow up medical treatments, therapy based on acceptance, commitment and hope therapy can be used to improve depression and death anxiety in patients with type 2 diabetes, thus preventing the complications of the disease

    Keywords: depression, Death anxiety, Hope-Based Therapy, Acceptance, Commitment Based Therapy, Diabetes
  • Sare Khodaee, Zahra Yousefi *, Floor Khayatan Pages 340-355
    Aim

    Nowadays, attention seeking has become a matter of concern in the family context and the need for prevention and adjustment is felt as a therapeutic necessity. Therefore, the present study aimed to investigate the components of effective management of attention seeking among married women from the perspective of family counselors and to present a treatment model.

    Methods

    From experienced counselors who had the necessary experience in the field of family therapy, using purposive sampling method, 8 people were selected to participate in the study and were interviewed. The study method was reflective theme analysis. The interview tool was a semi-structured interview.

    Results

    Data were analyzed by Clark and Brown method in six steps. The results of the analysis of the interviews showed that attention management is one of the three main categories of evaluation (with three organizing categories: self-report evaluation, evaluating symptoms with the help of interviews, and evaluating the individual and couple effects of attention seeking); 2) Individual therapy (with the organizing categories: insight, modifying cognitive stimulus cognitions, modifying attention-seeking behaviors and emotions) and couple therapy)with organizing categories: improving attachment, improving self-differentiation, and improving Life story) is composed.

    Conclusion

    According to the results of a three-dimensional treatment model of evaluation, individual and couple therapy was formed, which can be helpful and inspiring counselors interested in this field

    Keywords: Attention seeking, Family, Married women, Thematic Analysis, Therapeutic model
  • Pendar Jahedataeian, Hassan Shamsesfandabad *, Abdulhassan Farhangi Pages 356-369
    Aim

    The aim of this research was the effectiveness of positive psychology on self-compassion and gratitude among couples with marital conflicts.

    Method

    This research was practical in terms of purpose, and in terms of data collection, it was semi-experimental with a pre-test-post-test design with a control group. The statistical population of the research included couples with marital problems in Tehran who had referred to Tehran counseling centers (Tara Clinic Branches 1 and 2 and Behjo Clinic) in 2022 (184 couples), among them 30 couples with the condition of having the conditions of the research. and willingness to participate in the research were selected and randomly assigned to an experimental group and a control group and 15 couples in each group. Rashid and Seligman's protocol including 10 sessions of 90 minutes (2013) was used for positive psychology therapeutic interventions. The research tools included the Neff Self-Deprecation Questionnaire (2003) and the McCliffe Appreciation Questionnaire (2002). The data was analyzed using analysis of covariance using SPSS-V26 software.

    Results

    The findings showed that in the post-test stage, there is a significant difference between the experimental group and the control group (P<0.001) and the intervention method can be effective on the self-compassion (F=32.14, P<0.001) and gratitude (F=7.64, P<0.001) of the couples involved.

    Conclusion

    According to the results of the research, it can be stated that positive psychology as an effective treatment method can be used to improve, self-compassion and gratitude and consequently reduce the marital problems of couples, and it is recommended to psychologists and couple therapy specialists to use this treatment method for Use to improve relationships between couples

    Keywords: self-compassion, Gratitude, Positive psychology
  • Zahra Ismailian, Mohammadali Rahmani *, Taher Tizdast Pages 370-382
    Aim

    The purpose of this study was to determine the effectiveness of positive psychotherapy on adherence to treatment and glycosylated hemoglobin in female patients with type 2 diabetes.

    Method

    This study was a quasi-experimental type of pre-test, post-test and follow-up. 30 women with type 2 diabetes who referred to Iran Diabetes Association located in Tehran province from January to May 2019 were selected by purposive sampling method. Then they were randomly assigned to the experimental group (under positive psychotherapy) (n=15) and the control group (n=15). Data collection tools were the adherence to treatment scale (Murisky, Eng, & Wood, 2008) and glycosylated hemoglobin measurement in three baseline stages, after the intervention and three-month follow-up. The structure of positive psychotherapy sessions was implemented based on Seligman's positive psychotherapy group intervention (2006) in eight sessions for the experimental group. Data analysis was done using multivariate repeated measure analysis of variance.

    Results

    The findings showed that in the group under positive psychotherapy, the average score of adherence to treatment is significantly higher than the control group (F=511.16, P<0.001). In addition, women with diabetes in the experimental group decreased significantly in the average glycosylated hemoglobin score compared to the control group (F=389.50, P<0.001).

    Conclusion

    The results of this study showed that positive psychotherapy can be effective in improving treatment compliance and reducing glycosylated hemoglobin in women with type 2 diabetes.

    Keywords: Adherence to treatment, Type 2 diabetes, Positive psychotherapy, Glycosylated hemoglobin
  • Leila Asadi, Mohammadreza Seirafi *, Morvarid Ahadi Pages 383-400
    Aim

    The aim of this study was to determine the effectiveness of painting therapy on health anxiety, loneliness and death anxiety in the elderly.

    Methods

    The research method was quasi-experimental with pre-test-post-test design with experimental group and control group with a follow-up test period. From the statistical population of the elderly living in nursing homes in Tehran in 2021, 30 samples were selected by convenience sampling and randomly divided into two experimental groups and a control group. Painting therapy was applied for ten sessions for the experimental group. The required findings were collected using the Health Anxiety Questionnaire (Salkovskis & Warwick, 2002), Loneliness (Dehshiri et al., 2007) and Templer Death Anxiety (1970) and were analyzed using repeated measures analysis of variance was analyzed.

    Results

    Findings showed that the mean scores of post-tests and follow-up test of health anxiety, loneliness and death anxiety in the experimental group decreased and the results of repeated measures analysis of variance showed that painting therapy intervention to reduce health anxiety, loneliness and death anxiety in the elderly is effective (p <0.001).

    Conclusion

    Therefore, it is suggested that painting therapy intervention be used to reduce health anxiety, loneliness and death anxiety in the elderly

    Keywords: Elderly, Health Anxiety, Loneliness, Death anxiety, painting therapy
  • Sara Makipour, Abdollah Shafie Abadi *, Mohamadreza Falsafinejad, Anahita Khodabakhshi-Koolaee Pages 401-425
    Aim

    The purpose of this research was the effectiveness of family enrichment model for work on marital satisfaction, job satisfaction and work-family conflict of married female managers.

    Methods

    The present study was a semi-experimental study with an experimental and control group and a pre-test-post-test design with a six-month follow-up. The statistical population included all 150 married female managers working in the headquarters department of Persian Bank in Tehran in 2021, who along with their spouses were willing to participate in training sessions. The number of sample members was 30 couples who were replaced by random sampling in two experimental and control groups (15 couples in the experimental group and 15 couples in the control group). The experimental group was subjected to the intervention of the family enrichment model for the work of Makipour et al (2021) for 9 sessions of 90 minutes. Both groups were evaluated in the three stages of pre-test, post-test and follow-up by Enrich marital satisfaction questionnaires (1989), Minnesota job satisfaction scale (1943) and Rostgarkhald's work-family conflict scalhe (2002). Data were analyzed using repeated measures mixed analysis of variance.

    Results

    The results indicate the effect of family enrichment intervention for work on the score of marital satisfaction (P < 0.05, F = 6.07) and job satisfaction (P < 0.05, F = 60.32) in female managers, as well as the score Work-family conflict (P < 0.05, F = 8.17), marital satisfaction (P < 0.05, F = 18.53) and job satisfaction (P < 0.05, F = 69.72) in Spouses and lack of effect of family enrichment intervention for work on work-family conflict score (P < 0.05, F = 0.95) in female managers and the stability of these results was in the follow-up phase.

    Conclusion

    Considering the results obtained from the research, organizational managers can benefit from the educational model of family enrichment for work as an effective intervention method to improve the marital and job satisfaction of married female managers and couples and reduce the work-family conflict of both working couples.

    Keywords: : Family-to-work Enrichment Model, marital satisfaction, Job Satisfaction, work-family conflict, Women
  • Paria Mozhgani, Omid Moradi *, Hamzeh Ahmadian, Yahya Yarahmadi Pages 426-443
    Aim

    The aim of the present study was to compare the effectiveness of Psychodrama Couple Therapy and Integrated Behavioral Couple Therapy (IBCT) on improving conflict resolution tactics of contradictory couples.

    Method

    The method of this semi-experimental study is pre-test/post-test with a control group and a follow-up phase. The statistical population included all conflicting couples who referred to Dramatherapy Center of Iran in 2019. The sample of this study includes 36 persons (18 couples) who referred to Dramatherapy Center of Iran who were selected by the available sampling method. Then they were assigned in 3 groups of 12 persons including two experimental groups and one control group. The first experimental group underwent 12 sessions of Psychodrama Couple Therapy by Moreno (1969) and the second experimental group underwent 12 sessions of Integrative Behavioral Couple Therapy by Christensen et al. (2014). Data were obtained by using Conflict Tactics Scale (CTS2) by Straus (1996) and analyzed by methods of repeated measure analysis of variance using spss22 software.

    Results

    The results of this study showed that Psychodrama Couple Therapy reduced physical assault (F=11.52 and p=0.001) and psychological violence (F=12.74 and p=0.001) and increased non-violent negotiation (F=58.84 and p=0.001) in the victim form and respectively physical assault (F=17.96 and p=0.001), psychological violence (F=17.08 and p=0.001) and non-violent negotiation (F=89.45 and p=0.001) in the aggressive form.

    Conclusion

    The results indicated the effectiveness of Psychodrama Couple Therapy on reducing physical assault and psychological violence and increasing non-violent negotiation in conflicting couples and its durability over time. Therefore, health professionals and therapists can use Psychodrama Couple Therapy along with other treatment methods to improve couples' conflict resolution tactics.

    Keywords: Conflict Resolution Tactics, Couple Therapy, Psychodrama, Integrated Behavioral
  • Marjan Sadeghkhani, Suzan Emamipour *, Malek Mirhashemi Pages 444-471
    Aim

    The present study was conducted with the aim of modeling the structural equations of emotional divorce based on dark personality traits and conflict resolution styles with the mediation of marital dissatisfaction.

    Method

    The research method was descriptive correlation type based on structural equations. The statistical population of this research consisted of all married women in the age group of 25-50 years of Tehran Azad University, Yadgar Imam (RA), Abol Fattouh Razi (Mothers) and North Tehran units in the first half of 2021, which was available online through the sampling method. (due to the spread of the corona virus) were chosen. The research tools were Gottman's Emotional Divorce Questionnaire (1994), Dark Triangle of Personality Johnson and Webster (2010), Rahim's Conflict Resolution Styles (1983) and Pines Marital Boredom (1996). Analyzes were done using SPSS and AMOS software.

    Result

    The results of structural equation modeling showed that the assumed research model fits the experimental data (RMSEA=0.063). A significant relationship was observed between Machiavellian trait and emotional divorce (β=0.149) and between constructive style (β=0.441) and non-constructive style (β=0.264) with emotional divorce. The coefficient of the indirect path of marital boredom was significant in the relationship between narcissistic trait (P = 0.007) and constructive conflict resolution styles (P = 0.043) with the dependent variable of emotional divorce, but the mediating role of marital boredom in the relationship between non- constructive was not confirmed by emotional divorce (P = 0.536).

    Conclusion

    Dark traits and constructive conflict resolution styles, in addition to having a direct effect on emotional divorce, can indirectly affect marital boredom

    Keywords: conflict resolution styles, Dark personality traits, emotional divorce, marital boredom
  • Zohreh Zadhasan, Kobra Kazemian Moghadam * Pages 472-486
    Aim

    this study aimed to determine the effectiveness of emotion-focused couple therapy on positive meta-emotion, marital intimacy and personality abilities of elderly women.

    Methods

    The research method was semi-experimental with pre-test-posttest design with control group, and the statistical population of the study was all elderly women referred to counseling centers in Babol in 2021. The sample size consisted of 30 elderly women who were selected by convenience sampling methods according to inclusion and exclusion criteria. The experimental group received couple therapy intervention in 12 sessions of 90 minutes weekly, questionnaires used in this study included positive meta-emotion questionnaire of Mitmansgruber et al.(2009), Thompson and Walker marital intimacy (1993), and personality abilities of Peterson and Seligman (2004). Data were analyzed by repeatedly measuring variance in SPSS software version 22.

    Results

    The results showed that there was a significant difference between the experimental and control groups in terms of positive meta-emotion, marital intimacy and personality abilities (p<0.01). In other words, emotion-focused couple therapy intervention improved positive meta-emotion, marital intimacy and personality abilities of elderly women.

    Conclusion

    According to the findings of the present study, emotional-focused couple therapy intervention can be suggested as an efficient method to increase positive meta-emotion, marital intimacy and personality abilities of elderly women

    Keywords: Emotion-focused Couple Therapy, positive meta-emotion, Marital Intimacy, personality abilities, Elderly Women
  • Parisa Khatibi, Parisa Meghrazi * Pages 487-506
    Aim

    The aim of this study was to predict psychological distress based on initial maladaptive schemas and coping strategies in women affected by marital infidelity.

    Method

    The present study is descriptive-correlational in terms of method. The statistical population included all women involved in the issue of marital infidelity (spouse) who referred to counseling centers in District 5 of Tehran in 2021. 210 people were selected by convenience sampling method. Data collection tools included the Yeniseri and Kokdemir (2006) Marital Infidelity Questionnaire, the Lovibond & Lovibond (1995) Psychological Assistance Questionnaire, Lazarus Folkman (2001), and Young's Early maladaptive Schema Questionnaire (2000). Statistical analysis was performed using Pearson correlation coefficient and multiple regression method.

    Results

    The results of the study showed that based on the initial maladaptive schema and coping strategies, psychological distress in women affected by marital infidelity can be predicted.

    Conclusion

    The study of these predictor variables in premarital and family counseling is necessary to identify people at risk.

    Keywords: Psychological helplessness, early maladaptive schema, coping strategies
  • Mehdi Ahmadi, Saeed Eslami *, Alimohammad Mazidi sharafabadi Pages 507-529
    Aim

     The purpose of the current research was to develop a model of the functions of social networks in the control of the psychological security crisis of the family (the post-crisis phase of the Khuzestan flood case study). This research was done in two parts, qualitative and quantitative. 

    Method

     in the qualitative part, by interview and grounded theory method, the components of the model of social network functions effective on the control of the psychological security crisis of the family in the post-crisis phase were categorized into 6 data networks of the foundation, including: The central phenomenon, causal conditions, contextual conditions, intervening conditions, strategies and consequences were categorized. The statistical population in the qualitative part includes academic professors specializing in crisis management and the functions of social networks, as well as experienced experts in the field of flood crisis management in Khuzestan province who are active in this field (such as managers of the crisis management organization). Sampling method in the qualitative part was obtained from saturated and targeted level equal to 20 people. In the quantitative part, the qualitative model presented in the previous part was tested by questionnaire and structural equation method. The statistical population in the quantitative part was the experts and employees of the crisis management organization in Iran. 

    Findings

     Sampling in quantitative part was obtained by simple random sampling equal to 384 people. Based on the results of the qualitative section, the identified codes were categorized into 6 main networks, 16 main components and 87 sub-components. Based on the results of the quantitative part, causal conditions with a factor load of 0.588 have an effect on the central phenomenon. The central phenomenon, contextual conditions, and intervening conditions are effective on strategies with factor loadings of 0.515, 0.309, and 0.419, respectively, and strategies are effective on outcomes with factor loadings of 0.558. 

    Conclusion

     Based on the obtained results, it is suggested that in the post-crisis phase such as Khuzestan flood, the internet should be provided to the people of the society so that they can receive news and information quickly through social networks. Society must be ready to accept social networks; Therefore, for this purpose, the government should create the necessary culture for using social networks among the people.

    Keywords: Functions of social networks, crisis control, psychological security of the family
  • Sara Abolhasani, shohreh ghorban shiroudi *, mohammadreza zarbakhsh bahri Pages 561-564
    Aim

     This research aimed to determine the effectiveness of integrated systemic couple therapy on differentiation and marital disillusionment in couples affected by extramarital relationships. 

    Method

     The study employed a quasi-experimental design with pre-test, post-test, and follow-up stages, along with a control group. The statistical population consisted of women affected by extramarital relationships who sought counseling in Babol city centers during the first nine months of 2019. Thirty individuals were selected through convenience sampling and randomly assigned into two groups of fifteen for the experimental and control groups, respectively. The experimental group underwent Feldman's (1990) integrated systemic couple therapy protocol in twelve 90-minute group sessions, while the control group did not receive any training. Data were collected using the Pines' (1996) Marital Boredom Scale and the Skowron and Friedlander (1989) Differentiation of Self Inventory. Data were analyzed using repeated measures tests. 

    Results

     Results indicated that integrated systemic couple therapy had a significant effect on differentiation (F = 30.01, p < 0.001) and marital disillusionment (F = 78.86, p < 0.001) in women affected by extramarital relationships at the post-test stage. Additionally, the effectiveness of integrated systemic couple therapy on differentiation and marital disillusionment was sustained at the follow-up stage (p < 0.05). 

    Conclusion

     Given the findings, this research suggests that group-based integrated systemic couple therapy can aid in improving differentiation and marital disillusionment in women affected by extramarital relationships.

    Keywords: Integrated systemic couple therapy, differentiation, marital dissatisfaction, extramarital relations
  • Fateme Azad sarvestani *, Mahbobe Mina Pages 630-645
    Aim

     The purpose of the present study is to review some of the provisions of the Family Protection Act of 2012 and civil law to prevent the collapse of the family in a case whose subject is the sexual defects. 

    Method

     The present study uses a descriptive-analytical method and has been written using judicial opinions, books, dissertations and valid articles. 

    Results

     According to the findings of this study, in order to strengthen the family's foundation, couples' sexual disorders must be examined before marriage registration, and the dissolution of marriage due to termination and divorce should also be required to reform their couples. Treatment. In order to protect the rights of the wife under certain circumstances, although the couple's defect is treatable, the right to terminate the marriage for the wife remains a fault, and by providing sexual disorders in Article 6 of the Civil Code, the difficulty of proving the spouse shall be abolished. 

    Conclusion

     Sexual disorders that are treatable should not be considered as the dissolution of marriage, and the court should refer the couples to specialized centers for treatment. Reviewing some of the provisions of the Civil Code and the Family Protection Act of 2012 can cause timely diagnosis and treatment of couples' sexual disorders and reduce the number of marriage dissolution statistics.

    Keywords: Divorce at the request of the wife, Divorce under the law of Islamic countries, Sexual defects (disorders), Termination of marriage